Top-Rated Free Essay
Preview

Oncology Study Guide

Good Essays
22579 Words
Grammar
Grammar
Plagiarism
Plagiarism
Writing
Writing
Score
Score
Oncology Study Guide
Question 1:
(see full question)
Which of the following observations should the nurse make first when the client who has had a modified radical mastectomy returns from the operating room to the recovery room?
You selected:
Ensuring that the client’s airway is free of obstruction.
Correct

Explanation:
The highest priority when a nurse receives a client from the operating room is to assess airway patency. If the airway is not clear, immediate steps should be taken so that the cli... (more)
The highest priority when a nurse receives a client from the operating room is to assess airway patency. If the airway is not clear, immediate steps should be taken so that the client is able to breathe. Vital signs can be assessed after airway patency is assured. Assessing the patency and functioning of drainage tubes can be done after the airway is assessed and vital signs are taken. The dressing can be assessed once airway patency has been determined. (less)

Question 2:
(see full question)
The nurse is preparing a community presentation on oral cancer. Which of the following is a primary risk factor for oral cancer that the nurse should include in the presentation?
You selected:
Use of alcohol.
Correct

Explanation:
Chronic and excessive use of alcohol can lead to oral cancer. Smoking and use of smokeless tobacco are other significant risk factors. Additional risk factors include chronic irrit... (more)
Chronic and excessive use of alcohol can lead to oral cancer. Smoking and use of smokeless tobacco are other significant risk factors. Additional risk factors include chronic irritation such as a broken tooth or ill-fitting dentures, poor dental hygiene, overexposure to sun (lip cancer), and syphilis. Use of mouthwash, lack of vitamin B12, and lack of regular teeth cleaning appointments have not been implicated as primary risk factors for oral cancer. (less)

Question 3:
(see full question)
The nurse is instructing a client on how to care for skin that has become dry after radiation therapy. Which of the following statements by the client indicates that the client understands the teaching?
You selected:
"It is safe to apply a nonperfumed lotion to my skin."
Correct

Explanation:
Irradiated skin can become dry and irritated, resulting in itching and discomfort. The client should be instructed to clean the skin gently and apply nonperfumed, nonirritating lot... (more)
Irradiated skin can become dry and irritated, resulting in itching and discomfort. The client should be instructed to clean the skin gently and apply nonperfumed, nonirritating lotions to help relieve dryness. Taking an antihistamine does not relieve the skin dryness that is causing the itching. Heat should not be applied to the area because it can cause further irritation. Medicated ointments, especially corticosteroids, which is controversial, should not be applied to the skin without the order of the radiation therapist. (less)

Question 4:
(see full question)
A nurse is administering daunorubicin (Daunoxome/Cerubidine) through a peripheral I.V. line when the client complains of burning at the insertion site. The nurse notes no blood return from the catheter and redness at the I.V. site. The client is most likely experiencing which complication?
You selected:
Extravasation
Correct

Explanation:
The client is exhibiting signs of extravasation, which occurs when the medication leaks into the surrounding tissues and causes swelling, burning, or pain at the injection site. Er... (more)
The client is exhibiting signs of extravasation, which occurs when the medication leaks into the surrounding tissues and causes swelling, burning, or pain at the injection site. Erythema is redness of the skin that results from skin irritation. Flare is a spreading of redness that occurs as a result of drawing a pointed instrument across the skin. Thrombosis is the formation of clot within the vascular system. (less)

Question 5:
(see full question)
A 42-year-old female is interested in making dietary changes to reduce her risk of colon cancer. What dietary selections should the nurse suggest?
You selected:
Granola, bagel with cream cheese, cauliflower salad.
Incorrect

Correct response:
Bran muffin, skim milk, stir-fried broccoli.
Explanation:
High-fiber, low-fat diets are recommended to reduce the risk of colon cancer. Stir-frying, poaching, steaming, and broiling are all low-fat methods to prepare foods. Croissants are... (more)
High-fiber, low-fat diets are recommended to reduce the risk of colon cancer. Stir-frying, poaching, steaming, and broiling are all low-fat methods to prepare foods. Croissants are made of refined flour. They are also high in fat, as are peanut butter squares and whole milk, granola, cream cheese, and sour cream. (less)

Question 6:
(see full question)
Postoperatively after a modified radical mastectomy, a client has an incisional drainage tube attached to Hemovac suction. The nurse determines the suction is effective when:
You selected:
Accumulated serum and blood in the operative area are removed.
Correct

Explanation:
A drainage tube is placed in the wound after a modified radical mastectomy to help remove accumulated blood and fluid in the area. Removal of the drainage fluids assists in wound h... (more)
A drainage tube is placed in the wound after a modified radical mastectomy to help remove accumulated blood and fluid in the area. Removal of the drainage fluids assists in wound healing and is intended to decrease the incidence of hematoma, abscess formation, and infection. Drainage tubes placed in a wound do not decrease intrathoracic pressure, increase collateral lymphatic flow, or prevent adhesion formation. (less)

Question 7:
(see full question)
A client has just begun treatment with busulfan (Myleran/Busulfex), 4 mg by mouth daily, for chronic myelogenous leukemia. The client receives busulfan until his white blood cell (WBC) count falls to between 10,000/mm3 and 25,000/mm3. Then the drug is stopped. When should treatment resume?
You selected:
When the WBC count rises to 50,000/mm3
Correct

Explanation:
Busulfan treatment should resume when the WBC count rises to 50,000/mm3. Hair growth and anemia aren't appropriate markers for resuming busulfan treatment.... (more)
Busulfan treatment should resume when the WBC count rises to 50,000/mm3. Hair growth and anemia aren't appropriate markers for resuming busulfan treatment. (less)

Question 8:
(see full question)
A 52-year-old male was discharged from the hospital for cancer-related pain. His pain appeared to be well controlled on the I.V. morphine. He was switched to oral morphine when discharged 2 days ago. He now reports his pain as an 8 on a 10-point scale and wants the I.V. morphine. Which of the following represents the most likely explanation for the client's reports of inadequate pain control?
You selected:
He is undermedicated on the oral opioid.
Correct

Explanation:
Most cancer clients with inadequate pain control while taking an oral opioid after being switched from I.V. administration have been undermedicated. Equianalgesic conversions shoul... (more)
Most cancer clients with inadequate pain control while taking an oral opioid after being switched from I.V. administration have been undermedicated. Equianalgesic conversions should be made to provide estimates of the equivalent dose needed for the same level of relief as provided by the I.V. dose. There is research to suggest that cancer clients do not become addicted to opioids when dosed adequately. There is no evidence to suggest that the client is physically addicted or is having withdrawal symptoms. (less)

Question 9:
(see full question)
A client receiving 5-fluorouracil is experiencing nausea and vomiting. Which is the nurse’s best course of action?
You selected:
Administer odansetron (Zofran) prior to administering the 5-fluorouracil
Correct

Explanation:
Fluorouracil, an antimetabolite antineoplastic medication, may cause nausea, vomiting, diarrhea, bone marrow suppression, and stomatitis. Premedication with an antiemetic medicatio... (more)
Fluorouracil, an antimetabolite antineoplastic medication, may cause nausea, vomiting, diarrhea, bone marrow suppression, and stomatitis. Premedication with an antiemetic medication such as odansetron will prevent nausea and vomiting during treatment. (less)

Question 10:
(see full question)
Which of the following individuals is most at risk for acquiring acute lymphocytic leukemia (ALL)? The client who is:
You selected:
4 to 12 years.
Correct

Explanation:
The peak incidence of ALL is at 4 years of age. ALL is uncommon after 15 years of age. The median age at incidence of CML is 40 to 50 years. The peak incidence of AML occurs at 60 ... (more)
The peak incidence of ALL is at 4 years of age. ALL is uncommon after 15 years of age. The median age at incidence of CML is 40 to 50 years. The peak incidence of AML occurs at 60 years of age. Two-thirds of cases of chronic lymphocytic leukemia occur in clients older than 60 years of age. (less)

Question 11:
(see full question)
A nurse is caring for a client who is receiving chemotherapy and has a platelet count of 30,000/mm3. Which statement by the client indicates a need for additional teaching?
You selected:
"I use an electric razor to shave."
Incorrect

Correct response:
"I floss my teeth every morning."
Explanation:
A client with a platelet count of 30,000/mm3 is at risk for bleeding and shouldn't floss his teeth. Flossing may increase the risk of bleeding in a client with a platelet count less... (more)
A client with a platelet count of 30,000/mm3 is at risk for bleeding and shouldn't floss his teeth. Flossing may increase the risk of bleeding in a client with a platelet count less than 40,000/mm3. Using an electric razor is appropriate because doing so helps minimize the risk of cutting when shaving. Taking a stool softener helps decrease potential trauma to the GI tract that may cause bleeding. Removing throw rugs from the house helps prevent falls, which could lead to uncontrolled bleeding. (less)

Question 12:
(see full question)
On discharge, a client who underwent left modified radical mastectomy expresses relief that "the cancer" has been treated. When discussing this issue with the client, the nurse should stress that she:
You selected: should continue to perform breast self-examination on her right breast.
Correct

Explanation:
Having breast cancer on her left side puts the client more at risk for cancer on the opposite side and chest wall. Therefore, the nurse should stress the importance of monthly brea... (more)
Having breast cancer on her left side puts the client more at risk for cancer on the opposite side and chest wall. Therefore, the nurse should stress the importance of monthly breast self-examinations and annual mammograms. Although the tumor was found, it was large enough to require a mastectomy, and could put the patient at risk for metastasis. Follow-up appointments should be monthly for the first few months and then scheduled at the direction of her health care provider. Modified radical mastectomy shouldn't affect the menstrual cycle. (less)

Question 13:
(see full question)
Which of the following is an environmental factor that increases the risk of cancer?
You selected:
Immunologic status.
Incorrect

Correct response:
Nutrition.
Explanation:
Environmental factors include place of residence, nutrition, occupation, personal habits, iatrogenic factors, and physical environment. Gender, immunologic status, and age are indi... (more)
Environmental factors include place of residence, nutrition, occupation, personal habits, iatrogenic factors, and physical environment. Gender, immunologic status, and age are individual factors. (less)

Question 14:
(see full question)
A 3-year-old child receiving chemotherapy after surgery for a Wilms' tumor has developed neutropenia. The parent is trying to encourage the child to eat by bringing extra foods to the room. Which food would not be appropriate for this child?
You selected:
A milk shake.
Incorrect

Correct response:
Fresh strawberries.
Explanation:
When a client receiving chemotherapy develops neutropenia, eating uncooked fruits and vegetables may pose a health risk due to possible bacterial contamination. All other foods are... (more)
When a client receiving chemotherapy develops neutropenia, eating uncooked fruits and vegetables may pose a health risk due to possible bacterial contamination. All other foods are either cooked or pasteurized and would not produce a health risk. (less)

Question 15:
(see full question)
A client had a total abdominal hysterectomy and bilateral oophorectomy for ovarian carcinoma yesterday. She received 2 mg of morphine sulfate I.V. by patient-controlled analgesia (PCA) 10 minutes ago. The nurse was assisting her from the bed to a chair when the client felt dizzy and fell into the chair. The nurse should:
You selected:
Take the client's blood pressure.
Correct

Explanation:
The nurse should take the client's blood pressure. She is likely experiencing orthostatic hypotension. The PCA pump does not need to be discontinued because, as soon as the blood p... (more)
The nurse should take the client's blood pressure. She is likely experiencing orthostatic hypotension. The PCA pump does not need to be discontinued because, as soon as the blood pressure stabilizes, the pain medication can be resumed. Administering oxygen is not necessary unless the oxygen saturation also drops. The client should sit in the chair until the blood pressure stabilizes. (less)

Question 16:
(see full question)
A client undergoing chemotherapy has a white blood cell count of 2300/mm3 (2.3 X 109/L); hemoglobin of 9.8 g/dl (98 g/L); platelet count of 80,000/mm3 (80 X 109/L); and potassium of 3.8. Which of the following should take priority?
You selected:
Temperature 101° F (38.3° C).
Correct

Explanation:
The client has a low white blood cell count from the chemotherapy and has a temperature. Signs and symptoms of infection may be diminished in a client receiving chemotherapy; there... (more)
The client has a low white blood cell count from the chemotherapy and has a temperature. Signs and symptoms of infection may be diminished in a client receiving chemotherapy; therefore, the temperature elevation is significant. Early detection of the source of infection facilitates early intervention. Surveillance for bleeding is important with the low hemoglobin and platelet count, however, the high blood pressure does not indicate bleeding. Vomiting is a side effect of chemotherapy and should be treated. The urine output and potassium are within normal limits. (less)

Question 17:
(see full question)
The nurse has reinforced a pressure dressing on a client who is post-operative mastectomy and notes there is considerable sanguineous drainage in the hemovac. Which of the following assessments should the nurse report to the physician? Select all that apply.
You selected:
• Fever of 102F, no urine output for 2 hours
• Blood pressure 86/50 mm Hg; pulse weak and thready at 120 bpm
Correct

Explanation:
A weak, thready pulse may be a sign of hemorrhage. A fever with no urine output may also indicate a problem with the kidneys and possible infection. Cold, clammy skin can indicate ... (more)
A weak, thready pulse may be a sign of hemorrhage. A fever with no urine output may also indicate a problem with the kidneys and possible infection. Cold, clammy skin can indicate compensation if the BP is adequate. Warm, dry skin with a pulse of 86 bpm is an expected outcome. (less)

Question 18:
(see full question)
A client with suspected lung cancer is scheduled for thoracentesis as part of the diagnostic workup. The nurse reviews the client's history for conditions that might contraindicate this procedure. Which condition is a contraindication for thoracentesis?
You selected:
A bleeding disorder
Correct

Explanation:
A bleeding disorder is a contraindication for thoracentesis because a hemorrhage may occur during or after this procedure, possibly causing death. Although a history of a seizure d... (more)
A bleeding disorder is a contraindication for thoracentesis because a hemorrhage may occur during or after this procedure, possibly causing death. Although a history of a seizure disorder, COPD, or anemia calls for caution, it doesn't contraindicate thoracentesis. (less)

Question 19:
(see full question)
After cancer chemotherapy, a client experiences nausea and vomiting. The nurse should assign highest priority to which intervention?
You selected:
Administering metoclopramide and dexamethasone (Decadron) as ordered
Correct

Explanation:
The nurse should assign highest priority to administering an antiemetic, such as metoclopramide, and an anti-inflammatory agent, such as dexamethasone, because it may reduce the se... (more)
The nurse should assign highest priority to administering an antiemetic, such as metoclopramide, and an anti-inflammatory agent, such as dexamethasone, because it may reduce the severity of chemotherapy-induced nausea and vomiting. This intervention, in turn, helps prevent dehydration, a common complication of chemotherapy. Serving small portions of bland food, encouraging rhythmic breathing exercises, and withholding fluids for the first 4 to 6 hours are less likely to achieve this outcome. (less)

Question 20:
(see full question)
Cancer prevalence is defined as:
You selected:
The number of persons with cancer at a given point in time.
Correct

Explanation:
The word prevalence in a statistical setting is defined as the number of cases of a disease present in a specified population at a given time.... (more)
The word prevalence in a statistical setting is defined as the number of cases of a disease present in a specified population at a given time. (less)

Question 21:
(see full question)
A 21-year-old client undergoes bone marrow aspiration at the clinic to establish a diagnosis of possible lymphoma. Which statement made by the client demonstrates proper understanding of discharge teaching? Select all that apply.
You selected:
• "I will take Tylenol (acetaminophen) for pain."
• "I can apply an ice pack or a cold compress to the puncture site."
• "I will not be able to play basketball for the next 2 days."
Correct

Explanation:
Acetaminophen (Tylenol) is a safer analgesic than aspirin in order to avoid bleeding. Contact sports or trauma to the site should be avoided. Cool compresses should limit swelling ... (more)
Acetaminophen (Tylenol) is a safer analgesic than aspirin in order to avoid bleeding. Contact sports or trauma to the site should be avoided. Cool compresses should limit swelling and bruising. The puncture site should be inspected every 2 hours for bleeding or bruising during the first 24 hours. (less)

Question 22:
(see full question)
A right orchiectomy is performed on a client with a testicular malignancy. The client expresses concerns regarding his sexuality. The nurse should base the response on the knowledge that the client:
You selected:
Should retain normal sexual drive and function.
Correct

Explanation:
Unilateral orchiectomy alone does not result in impotence if the other testis is normal. The other testis should produce enough testosterone to maintain normal sexual drive, functi... (more)
Unilateral orchiectomy alone does not result in impotence if the other testis is normal. The other testis should produce enough testosterone to maintain normal sexual drive, functioning, and characteristics. Sperm banking before treatment is commonly recommended because radiation or chemotherapy can affect fertility. (less)

Question 23:
(see full question)
A 62-year-old female is taking long-acting morphine 120 mg every 12 hours for pain from metastatic breast cancer. She can have 20 mg of immediate-release morphine every 3 to 4 hours as needed for breakthrough pain. The physician should be notified if the client uses more than how many breakthrough doses of morphine in 24 hours?
You selected:
Seven.
Correct

Explanation:
If the maximum dose specified by the physician's order is required every 3-4 hours for break-through pain, the physician should be notified to increase the long-acting medication o... (more)
If the maximum dose specified by the physician's order is required every 3-4 hours for break-through pain, the physician should be notified to increase the long-acting medication or rotate to another type of opioid. Around-the-clock dosing is mandatory to achieve a steady state of analgesia. The rescue dose for breakthrough pain is administered over and above the regularly scheduled medication. If three to four analgesic doses are required every 24 hours, the sustained-release around-the-clock dose should be increased to include the amount used for previous breakthrough pain while maintaining a dose for future breakthrough pain. (less)

Question 24:
(see full question)
As part of a primary cancer prevention program, an oncology nurse answers questions from the public at a health fair. When someone asks about laryngeal cancer, the nurse should explain that:
You selected: laryngeal cancer is one of the most preventable types of cancer.
Correct

Explanation:
Laryngeal cancer is one of the most preventable types of cancer; it can be prevented by abstaining from excessive drinking and smoking. Inhaling noxious fumes, such as in polluted ... (more)
Laryngeal cancer is one of the most preventable types of cancer; it can be prevented by abstaining from excessive drinking and smoking. Inhaling noxious fumes, such as in polluted air, is a risk factor for laryngeal cancer. Roughly 80% of laryngeal cancer cases occur in men. Squamous cell carcinoma accounts for most cases of laryngeal cancer. (less)

Question 25:
(see full question)
A client diagnosed with seminomatous testicular cancer expresses fear and questions the nurse about his prognosis. The nurse should base the response on the knowledge that:
You selected:
Testicular cancer has a cure rate of 90% when diagnosed early.
Correct

Explanation:
When diagnosed early and treated aggressively, testicular cancer has a cure rate of about 90%. Treatment of testicular cancer is based on tumor type, and seminoma cancer has the be... (more)
When diagnosed early and treated aggressively, testicular cancer has a cure rate of about 90%. Treatment of testicular cancer is based on tumor type, and seminoma cancer has the best prognosis. Modes of treatment include combinations of orchiectomy, radiation therapy, and chemotherapy. The chemotherapeutic regimen used currently is responsible for the successful treatment of testicular cancer. (less)

Question 26:
(see full question)
Before a cancer client receiving total parenteral nutrition resumes a normal diet, the nurse teaches him about dietary sources of minerals. Which foods are good sources of zinc?
You selected:
Whole grains and meats
Correct

Explanation:
Good sources of zinc include whole grains, meats, dairy products, and seafood. Fruits are good sources of vitamin C, and vegetables are a good source of many vitamins and minerals,... (more)
Good sources of zinc include whole grains, meats, dairy products, and seafood. Fruits are good sources of vitamin C, and vegetables are a good source of many vitamins and minerals, but not zinc. Yeast is a good source of chromium, and legumes are a good source of copper, manganese, and molybdenum. (less)

Question 27:
(see full question)
A client is to start chemotherapy to treat lung cancer. A venous access device is placed to administer chemotherapeutic medications. Three days later at the scheduled appointment to receive chemotherapy, the nurse assesses that the client is dyspneic and the skin is warm and pale. Vital signs are BP 80/30, P 132, R 28, T 103 degrees F (39.4 degrees C), and oxygen saturation 84%. The central line insertion site is inflamed. After calling the rapid response team, what should the nurse do next?
You selected:
Administer a prescribed antipyretic.
Incorrect

Correct response:
Insert a peripheral intravenous fluid line and infuse normal saline.
Explanation:
The client is experiencing severe sepsis, and it is essential to increase circulating fluid volume to restore blood pressure and cardiac output. Applying a wet compress, administer... (more)
The client is experiencing severe sepsis, and it is essential to increase circulating fluid volume to restore blood pressure and cardiac output. Applying a wet compress, administering an antipyretic, and monitoring the client’s cardiac status may be beneficial for this client, but they are not the highest priority action at this time. These three interventions may require the nurse to leave the client, which is not advisable at this time. (less)

Question 28:
(see full question)
A client with cancer has diarrhea and inflamed areas of skin around the rectum. The nurse should do which of the following? Select all that apply.
You selected:
• Clean the rectal area with unscented soap and water after each bowel movement, rinse well, and pat dry.
• Use sitz baths.
• Apply a skin-barrier dressing daily to the rectal area.
Incorrect

Correct response:
• Use sitz baths.
• Clean the rectal area with unscented soap and water after each bowel movement, rinse well, and pat dry.
• Increase fluid intake.
Explanation:
The rectal area needs to be cleaned and gently dried after each bowel movement to prevent skin breakdown and inhibit growth of bacteria. Sitz baths are appropriate because they pro... (more)
The rectal area needs to be cleaned and gently dried after each bowel movement to prevent skin breakdown and inhibit growth of bacteria. Sitz baths are appropriate because they promote comfort. The client should increase fluid consumption to prevent dehydration. Zinc oxide ointment does form a protective skin barrier, but it makes it difficult to thoroughly clean the perirectal area of feces and increases the risk of infection, as do skin-barrier dressings. (less)

Question 29:
(see full question)
The development of laryngeal cancer is most clearly linked to which of the following factors?
You selected:
Alcohol and tobacco use.
Correct

Explanation:
Predisposing factors for laryngeal cancer include chronic irritants such as alcohol, tobacco, and exposure to noxious fumes. About 75% of people who develop laryngeal cancer are sm... (more)
Predisposing factors for laryngeal cancer include chronic irritants such as alcohol, tobacco, and exposure to noxious fumes. About 75% of people who develop laryngeal cancer are smokers. The combination of smoking and heavy alcohol intake is even more strongly implicated as a causative agent in laryngeal cancer. Epidemiologic studies indicate that a high-fat diet may be a major factor in the development of cancer of the breast, prostate, and colon, but not laryngeal cancer. Low socioeconomic status is a predisposing factor in cervical cancer but not laryngeal cancer. Artificial sweeteners have been related to the incidence of bladder cancer, but not laryngeal cancer. (less)

Question 30:
(see full question)
A 56-year-old female is currently receiving radiation therapy to the chest wall for recurrent breast cancer. She has pain while swallowing and burning and tightness in her chest. The nurse should further assess the client for indications of:
You selected:
Esophagitis.
Correct

Explanation:
Difficulty in swallowing, pain, and tightness in the chest are signs of esophagitis, which is a common complication of radiation therapy of the chest wall. Hiatal hernia is a herni... (more)
Difficulty in swallowing, pain, and tightness in the chest are signs of esophagitis, which is a common complication of radiation therapy of the chest wall. Hiatal hernia is a herniation of a portion of the stomach into the esophagus. The client could experience burning and tightness in the chest secondary to a hiatal hernia, but not pain when swallowing. Also, hiatal hernia is not a complication of radiation therapy. Stomatitis is an inflammation of the oral cavity characterized by pain, burning, and ulcerations. The client with stomatitis may experience pain with swallowing, but not burning and tightness in the chest. Radiation enteritis is a disorder of the large and small bowel that occurs during or after radiation therapy to the abdomen, pelvis, or rectum. Nausea, vomiting, abdominal cramping, the frequent urge to have a bowel movement, and watery diarrhea are the signs and symptoms. (less)

Question 31:
(see full question)
Which of the following is an early sign of laryngeal cancer?
You selected:
Persistent mild hoarseness.
Correct

Explanation:
Hoarseness occurs early in the course of most laryngeal cancers because the tumor prevents accurate approximation of the vocal cords during phonation. Large tumors eventually produ... (more)
Hoarseness occurs early in the course of most laryngeal cancers because the tumor prevents accurate approximation of the vocal cords during phonation. Large tumors eventually produce difficulty and pain in swallowing, but this is not an early sign. Foul breath and expectoration of blood are late symptoms. A nagging cough has no direct relationship to laryngeal cancer. (less)

Question 32:
(see full question)
A young female client is receiving chemotherapy and mentions to the nurse that she and her husband are using a diaphragm for birth control. Which of the following is most important for the nurse to discuss?
You selected:
Infection control.
Correct

Explanation:
The risk of becoming neutropenic during chemotherapy is very high. Therefore, an inserted foreign object such as a diaphragm may be a nidus for infection. Although the nurse may wi... (more)
The risk of becoming neutropenic during chemotherapy is very high. Therefore, an inserted foreign object such as a diaphragm may be a nidus for infection. Although the nurse may wish to inform the client about the ease with which various contraceptive modalities may be used, the focus of this discussion should be on preventing an infection, which can be fatal for the neutropenic client. There are no data to suggest the client is at risk for acquiring a sexually transmitted disease. The client will not be experiencing body changes directly related to hormonal changes. (less)

Question 33:
(see full question)
A nurse is planning a program about cancer for a woman’s club. Which of the following should the nurse include in the discussion of risk factors for ovarian cancer?
You selected:
A 42-year-old woman who has never been pregnant
Correct

Explanation:
The incidence of cancer increases with age in women who have never been pregnant. Being an older female with a small frame and fair skin are risk factors for osteoporosis. Women wh... (more)
The incidence of cancer increases with age in women who have never been pregnant. Being an older female with a small frame and fair skin are risk factors for osteoporosis. Women who breast feed have a lower risk of breast and ovarian cancer. Oral contraceptive therapy reduces the risk of ovarian cancer. The risk is lower the longer the pills are used. This lower risk continues for many years after the pill is stopped. (less)

Question 34:
(see full question)
A client with a cerebellar brain tumor is admitted to an acute care facility. The nurse formulates a nursing diagnosis of Risk for injury. Which "related-to" phrase should the nurse add to complete the nursing diagnosis statement?
You selected:
Related to impaired balance
Correct

Explanation:
A client with a cerebellar brain tumor may suffer injury from impaired balance as well as disturbed gait and incoordination. Visual field deficits, difficulty swallowing, and psych... (more)
A client with a cerebellar brain tumor may suffer injury from impaired balance as well as disturbed gait and incoordination. Visual field deficits, difficulty swallowing, and psychomotor seizures may result from dysfunction of the pituitary gland, pons, occipital lobe, parietal lobe, or temporal lobe — not from a cerebellar brain tumor. Difficulty swallowing suggests medullary dysfunction. Psychomotor seizures suggest temporal lobe dysfunction. (less)

Question 35:
(see full question)
A nurse is assessing a client with multiple myeloma. The nurse should keep in mind that clients with multiple myeloma are at risk for:
You selected: pathologic bone fractures.
Correct

Explanation:
Clients with multiple myeloma are at risk for pathologic bone fractures secondary to diffuse osteoporosis and osteolytic lesions. Also, clients are at risk for renal failure second... (more)
Clients with multiple myeloma are at risk for pathologic bone fractures secondary to diffuse osteoporosis and osteolytic lesions. Also, clients are at risk for renal failure secondary to myeloma proteins by causing renal tubular obstruction. Liver failure and heart failure aren't usually sequelae of multiple myeloma. Hypoxemia isn't usually related to multiple myeloma. (less)

Question 36:
(see full question)
A nurse is caring for a client with bronchogenic carcinoma. Which nursing intervention takes highest priority?
You selected:
Removing pulmonary secretions
Correct

Explanation:
Maintaining a patent airway is the first concern in a client with a condition that may compromise the airway. Therefore, adequate removal of pulmonary secretions is a priority. Alt... (more)
Maintaining a patent airway is the first concern in a client with a condition that may compromise the airway. Therefore, adequate removal of pulmonary secretions is a priority. Although clients may exhibit fatigue, anxiety, or appetite loss, these need to be addressed, but are not the priority. (less)

Question 37:
(see full question)
A nurse is teaching a 62-year-old female who has had a left modified radical mastectomy with axillary node dissection about lymphedema. The nurse should tell the client that lymphedema occurs:
You selected:
At any time after surgery or not at all.
Correct

Explanation:
Lymphedema after breast cancer surgery is the accumulation of lymph tissue in the tissues of the upper extremity extending down from the upper arm. It may occur at any time after s... (more)
Lymphedema after breast cancer surgery is the accumulation of lymph tissue in the tissues of the upper extremity extending down from the upper arm. It may occur at any time after surgery in women of any age. It is caused by the interruption or removal of lymph channels and nodes after axillary node dissection. Removal results in less efficient filtration of lymph fluid and a pooling of lymph fluid in the tissues on the affected side. Treatments or interventions should be instituted as soon as lymphedema is noted to prevent or reduce further progression. Range-of-motion exercises, elevation, and avoidance of injury in the affected arm are important when completing client teaching. Lymphoma is not caused by failure to remove all cancer cells. Lymphedema can occur after any surgery that disrupts lymph flow, not just radical mastectomy. (less)

Question 38:
(see full question)
A client diagnosed with acute myelocytic leukemia has been receiving chemotherapy. During the last 2 cycles of chemotherapy, the client developed severe thrombocytopenia requiring multiple platelet transfusions. The client is now scheduled to receive a third cycle. How can the nurse best detect early signs and symptoms of thrombocytopenia?
You selected:
Closely observe the client's skin for petechiae and bruising.
Correct

Explanation:
The nurse should closely observe the client's skin for petechiae and bruising. Daily laboratory testing may not reflect the client's condition as quickly as subtle changes in the c... (more)
The nurse should closely observe the client's skin for petechiae and bruising. Daily laboratory testing may not reflect the client's condition as quickly as subtle changes in the client's skin. Performing a cardiovascular assessment every 4 hours and checking the clients history for a congenital link to thrombocytopenia don't help detect early signs and symptoms of thrombocytopenia. (less)

Question 39:
(see full question)
The nurse advises a 42-year-old client to have a screening mammogram. The client asks why this is necessary since she performs a breast self-examination (BSE) monthly. The nurse's best response is:
You selected:
"Most women don't perform BSE thoroughly enough to detect cancer."
Incorrect

Correct response:
"A mammogram can identify breast cancer before it's detectable by BSE."
Explanation:
A mammogram can detect a lesion the size of a pinhead, and a lump is about 2 cm before it can be detected by a BSE. The American and Canadian Cancer Societies guidelines recommend ... (more)
A mammogram can detect a lesion the size of a pinhead, and a lump is about 2 cm before it can be detected by a BSE. The American and Canadian Cancer Societies guidelines recommend a mammogram yearly after age 40. A mammogram will not detect other endocrine abnormalities. (less)

Question 40:
(see full question)
A client with bladder cancer has lost an estimated 500 ml blood in the urine. The client's hemoglobin is 8.0 g/dl (80 g/L), and the physician orders a unit of packed blood cells. To administer the packed red blood cells, the nurse should:
You selected:
Attach the packed blood cells to the existing 22G I.V. of 5% dextrose using Y tubing.
Incorrect

Correct response:
Attach the packed cells to the existing 19G I.V. of normal saline solution using Y tubing.
Explanation:
The packed cells should be administered using a central catheter or 19G needle. Y tubing is used and the normal saline solution is used to keep the vein open when the blood transfu... (more)
The packed cells should be administered using a central catheter or 19G needle. Y tubing is used and the normal saline solution is used to keep the vein open when the blood transfusion is complete. Blood is not compatible with dextrose because dextrose may cause blood coagulation. Blood products should be given with normal saline solution. A blood filter must be used for all blood products to filter out sediment from stored blood products. It is not necessary to add another I.V. access. (less)

Question 41:
(see full question)
For a client with newly diagnosed cancer, the nurse formulates a nursing diagnosis of Anxiety related to the threat of death secondary to cancer diagnosis. Which expected outcome is appropriate for this client?
You selected:
"Client verbalizes feelings of anxiety."
Correct

Explanation:
Verbalizing feelings is the client's first step in coping with the situational crisis. It also helps the health care team gain insight into the client's feelings, helping guide psy... (more)
Verbalizing feelings is the client's first step in coping with the situational crisis. It also helps the health care team gain insight into the client's feelings, helping guide psychosocial care. An outcome in which the client doesn't guess at the prognosis is inappropriate because suppressing speculation may prevent the client from coming to terms with the crisis and planning accordingly. An outcome in which the client uses any effective method to reduce tension is undesirable because some methods of reducing tension, such as illicit drug or alcohol use, may prevent the client from coming to terms with the threat of death as well as cause physiologic harm. An outcome in which the client stops seeking information isn't appropriate because seeking information can help a client with cancer gain a sense of control over the crisis. (less)

Question 42:
(see full question)
A client with cancer is receiving chemotherapy. The nurse should assess which of the following diagnostic values while the client is receiving chemotherapy?
You selected:
Bone marrow cells
Correct

Explanation:
The fast-growing, normal cells most likely to be affected by certain cancer treatments are blood-forming cells in the bone marrow, as well as cells in the digestive tract, reproduc... (more)
The fast-growing, normal cells most likely to be affected by certain cancer treatments are blood-forming cells in the bone marrow, as well as cells in the digestive tract, reproductive system, and hair follicles. Fortunately, most normal cells recover quickly when treatment is over. Bone marrow suppression (decreased ability of the bone marrow to manufacture blood cells) is a common side effect of chemotherapy. A low white blood cell count (neutropenia) increases the risk for infection during chemotherapy, but other blood cells made in the bone marrow can be affected as well. Most cancer agents do not affect tissues and organs, such as the heart, liver, and pancreas. (less)

Question 43:
(see full question)
The goal of nursing care for a client with acute myeloid leukemia (AML) is to prevent:
You selected:
Hemorrhage.
Correct

Explanation:
Bleeding and infection are the major complications and causes of death for clients with AML. Bleeding is related to the degree of thrombocytopenia, and infection is related to the ... (more)
Bleeding and infection are the major complications and causes of death for clients with AML. Bleeding is related to the degree of thrombocytopenia, and infection is related to the degree of neutropenia. Cardiac arrhythmias rarely occur as a result of AML. Liver or renal failure may occur, but neither is a major cause of death in AML. (less)

Question 44:
(see full question)
Several days before admission, a client reports finding a small lump in the left breast near the nipple. What should the nurse tell the client to do?
You selected:
Inform the physician immediately.
Correct

Explanation:
The client should notify the physician immediately because a breast lump may be a sign of breast cancer. The client shouldn't squeeze the nipple to check for drainage until the phy... (more)
The client should notify the physician immediately because a breast lump may be a sign of breast cancer. The client shouldn't squeeze the nipple to check for drainage until the physician examines the area. The client shouldn't wait until after the next menstrual period to inform the physician of the breast lump because prompt treatment may be necessary. Placing a heating pad on the area would have no effect on a breast lump. (less)

Question 45:
(see full question)
A client complains of sporadic epigastric pain, yellow skin, nausea, vomiting, weight loss, and fatigue. Suspecting gallbladder disease, the physician orders a diagnostic workup, which reveals gallbladder cancer. Which nursing diagnosis is appropriate for this client?
You selected:
Anticipatory grieving
Correct

Explanation:
Anticipatory grieving is an appropriate nursing diagnosis for this client because few clients with gallbladder cancer live more than 1 year after diagnosis. Impaired swallowing isn... (more)
Anticipatory grieving is an appropriate nursing diagnosis for this client because few clients with gallbladder cancer live more than 1 year after diagnosis. Impaired swallowing isn't associated with gallbladder cancer. Although surgery typically is done to remove the gallbladder and, possibly, a section of the liver, it isn't disfiguring and doesn't cause Disturbed body image. Chronic low self-esteem isn't an appropriate nursing diagnosis at this time because the diagnosis has just been made. (less)

Question 46:
(see full question)
Which statement by a client undergoing external radiation therapy indicates the need for further teaching?
You selected:
"I'm worried I'll expose my family members to radiation."
Correct

Explanation:
The client undergoing external radiation therapy requires further teaching when he voices a concern that he might expose his family to radiation. Internal radiation, not external r... (more)
The client undergoing external radiation therapy requires further teaching when he voices a concern that he might expose his family to radiation. Internal radiation, not external radiation, poses a risk to the client's family. The client requires no further teaching if he states that he should wash his skin with mild soap and water, wear protective clothing when outside, and avoid using a heating pad. (less)

Question 47:
(see full question)
A nurse is teaching a male client to perform monthly testicular self-examinations. Which point is appropriate to make?
You selected:
Testicular cancer is a highly curable type of cancer.
Correct

Explanation:
Testicular cancer is highly curable, particularly when it's treated in its early stage. Self-examination allows early detection and facilitates the early initiation of treatment. T... (more)
Testicular cancer is highly curable, particularly when it's treated in its early stage. Self-examination allows early detection and facilitates the early initiation of treatment. The highest mortality rates from cancer among men are in men with lung cancer. Testicular cancer is found more commonly in younger, not older, men. (less)

Question 48:
(see full question)
On the third postoperative day after a radical mastectomy, the drainage tube is removed, and the dressings are changed. The client appears shocked when she sees the operative area and exclaims, “I look horrible! Will it ever look better?” Which of the following responses by the nurse would be most appropriate?
You selected:
”You’re shocked by the sudden change in your appearance as a result of this surgery, aren’t you?”
Correct

Explanation:
When a client appears shocked by her appearance after surgery, such as after having a mastectomy, the nurse should help her express her feelings and offer the supportive care that ... (more)
When a client appears shocked by her appearance after surgery, such as after having a mastectomy, the nurse should help her express her feelings and offer the supportive care that she needs at this time. Telling the client that her disfigurement will not show when she is dressed dismisses her concerns and blocks expression of her feelings. Telling the client not to worry avoids the issues. Having the client meet someone who has had breast surgery is often helpful but is better done later, when the client is convalescing and accustomed to the appearance of the operative site. The client needs support now when the dressings are removed, not later. (less)

Question 49:
(see full question)
A client receiving chemotherapy for cervical cancer indicates that she has an advance directive. She tells the nurse that she worries her children will not honor her wishes if her condition should worsen. In order to facilitate the honoring of the client's wishes, what should the nurse encourage the client to do?
You selected:
Discuss her end-of-life wishes with her family.
Correct

Explanation:
Family opposition does not override an advance directive. However, the client should ensure that family members know what her wishes are, even if they do not agree with them. After... (more)
Family opposition does not override an advance directive. However, the client should ensure that family members know what her wishes are, even if they do not agree with them. After discussing her wishes with her family, the client can decide if she should seek additional legal advice, obtain legal documents, or name an outside proxy. (less)

Question 50:
(see full question)
A 32-year-old woman recently diagnosed with Hodgkin's disease is admitted for staging by undergoing a bone marrow aspiration and biopsy. To obtain more information about the client's nutrition status the nurse should review the results of which of the following tests?
You selected:
Albumin level.
Correct

Explanation:
Serum albumin levels help determine whether protein intake is sufficient. Proteins are broken down into amino acids during digestion. Amino acids are absorbed in the small intestin... (more)
Question 1:
(see full question)
A nurse is teaching a community class about how to decrease the risk of cancer. What is the best food for the nurse to recommend?
You selected:
Oranges
Correct

Explanation:
A diet high in vitamin C and citrus may help reduce the risk of certain cancers, such as stomach and esophageal cancers. Hot dogs, smoked and cured foods are high in nitrates, whic... (more)
A diet high in vitamin C and citrus may help reduce the risk of certain cancers, such as stomach and esophageal cancers. Hot dogs, smoked and cured foods are high in nitrates, which may be linked to esophageal and gastric cancers. The chemical process used to decaffeinate coffee contributes to cancer. (less)

Question 2:
(see full question)
A client with stage II ovarian cancer undergoes a total abdominal hysterectomy and bilateral salpingo-oophorectomy with tumor resection, omentectomy, appendectomy, and lymphadenectomy. During the second postoperative day, which assessment finding requires immediate intervention?
You selected:
Shallow breathing and increasing lethargy
Correct

Explanation:
Shallow breathing and a change in the level of consciousness, such as increasing lethargy requires immediate intervention because they may indicate a respiratory complication ̵... (more)
Shallow breathing and a change in the level of consciousness, such as increasing lethargy requires immediate intervention because they may indicate a respiratory complication — for example, atelectasis or carbon dioxide retention. To avoid respiratory complications, the nurse should encourage turning, coughing, deep breathing, and ambulation during the early postoperative period. Abdominal pain, hypoactive bowel sounds, and serous drainage from the incision are expected findings during the first few days after this type of surgery. (less)

Question 3:
(see full question)
A 36-year-old female is scheduled to receive external radiation therapy and a cesium implant for cancer of the cervix. Which of the following statements would be most accurate to include in the teaching plan about the potential effects of radiation therapy on sexuality?
You selected:
"You may notice some vaginal dryness after treatment is completed."
Correct

Explanation:
Radiation fields that include the ovaries usually result in premature menopause. Vaginal dryness will occur without estrogen replacement. There should be no sexual intercourse whil... (more)
Radiation fields that include the ovaries usually result in premature menopause. Vaginal dryness will occur without estrogen replacement. There should be no sexual intercourse while the implant is in place. Cesium is a radioactive isotope used for therapeutic irradiation of cancerous tissue. There is no documentation to support vaginal relaxation after treatment. Because the client will have premature menopause, she will not have normal menstrual periods. (less)

Question 4:
(see full question)
The nurse at the gynecologic clinic is teaching the client about the results of her Papanicolaou test, which demonstrated dysplasia. Which represents the nurse’s best intervention?
You selected:
Explain the results show alteration in the size and shape of cells, which requires follow-up.
Correct

Explanation:
Dysplasia, a precancerous condition, refers to an alteration in the size, shape, and organization of differentiated cells. The client will need further diagnostic evaluation to det... (more)
Dysplasia, a precancerous condition, refers to an alteration in the size, shape, and organization of differentiated cells. The client will need further diagnostic evaluation to determine scope and treatment of the problem. The other options are not the best interventions. (less)

Question 5:
(see full question)
A 56-year-old woman is admitted for a modified radical mastectomy. The client appears anxious and asks many questions. The nurse's best course of action is to:
You selected:
Tell the client as much as she wants to know and is able to understand.
Correct

Explanation:
An important nursing responsibility is preoperative teaching. The recommended guide for teaching is to tell the client as much as she wants to know and is able to understand. Delay... (more)
An important nursing responsibility is preoperative teaching. The recommended guide for teaching is to tell the client as much as she wants to know and is able to understand. Delaying discussion of issues or concerns will most likely increase the client's anxiety. Telling the client to discuss questions with the physician avoids acknowledging the client's concerns. (less)

Question 6:
(see full question)
Experimental and epidemiologic evidence suggests that a high-fat diet increases the risk of several cancers. Which of the following cancers is linked to a high-fat diet?
You selected:
Colon.
Correct

Explanation:
Evidence suggests that a high-fat diet increases the risk of several cancers, including breast, colon, and prostate cancers. Ovarian, lung, and liver cancers have not been linked t... (more)
Evidence suggests that a high-fat diet increases the risk of several cancers, including breast, colon, and prostate cancers. Ovarian, lung, and liver cancers have not been linked to a high-fat diet. (less)

Question 7:
(see full question)
A client is newly diagnosed with cancer and is beginning a treatment plan. Which of the following nursing interventions will be most effective in helping the client cope?
You selected:
Encourage strict compliance with all treatment regimens.
Incorrect

Correct response:
Identify available resources.
Explanation:
Identifying available resources for the client and family represents a respectful effort to make options available and encourages the client to become involved in treatment decisio... (more)
Identifying available resources for the client and family represents a respectful effort to make options available and encourages the client to become involved in treatment decisions. Assuming decision making for the client may foster dependence. Encouraging strict compliance with all treatment regimens may increase anxiety and limit the client's options and treatment choices. Informing the client of all possible adverse treatment effects may increase anxiety and fear by focusing on adverse outcomes too soon. (less)

Question 8:
(see full question)
A client is scheduled to undergo weekly intravesical chemotherapy for bladder cancer for the next 8 weeks. The nurse determines that the client understands how to manage the urine as a biohazard if the client tells the nurse he will:
You selected:
Disinfect the urine and toilet with bleach for 6 hours following a treatment.
Correct

Explanation:
After intravesical chemotherapy, the client must treat the urine as a biohazard; this involves disinfecting the urine and the toilet with household bleach for 6 hours following a t... (more)
After intravesical chemotherapy, the client must treat the urine as a biohazard; this involves disinfecting the urine and the toilet with household bleach for 6 hours following a treatment. It is not necessary to use a bedpan and then empty the urine in the toilet; the client can use the toilet, but must disinfect the urine with bleach. The bathroom does not need to be cleaned daily with disinfectant wipes. The client does not need to use a separate bathroom as long as the client's urine is disinfected with bleach. (less)

Question 9:
(see full question)
A client complains of sporadic epigastric pain, yellow skin, nausea, vomiting, weight loss, and fatigue. Suspecting gallbladder disease, the physician orders a diagnostic workup, which reveals gallbladder cancer. Which nursing diagnosis is appropriate for this client?
You selected:
Anticipatory grieving
Correct

Explanation:
Anticipatory grieving is an appropriate nursing diagnosis for this client because few clients with gallbladder cancer live more than 1 year after diagnosis. Impaired swallowing isn... (more)
Anticipatory grieving is an appropriate nursing diagnosis for this client because few clients with gallbladder cancer live more than 1 year after diagnosis. Impaired swallowing isn't associated with gallbladder cancer. Although surgery typically is done to remove the gallbladder and, possibly, a section of the liver, it isn't disfiguring and doesn't cause Disturbed body image. Chronic low self-esteem isn't an appropriate nursing diagnosis at this time because the diagnosis has just been made. (less)

Question 10:
(see full question)
A nurse is assessing a client with metastatic lung cancer. The nurse should assess the client specifically for:
You selected:
Hoarseness.
Correct

Explanation:
Hoarseness may indicate metastatic disease to the recurrent laryngeal nerve and is commonly noted with left upper lobe lung tumors. Diarrhea and constipation are not associated wit... (more)
Hoarseness may indicate metastatic disease to the recurrent laryngeal nerve and is commonly noted with left upper lobe lung tumors. Diarrhea and constipation are not associated with lung cancer. Weight loss, not weight gain, can be a symptom of extensive disease. (less)

Question 11:
(see full question)
After a transsphenoidal adenohypophysectomy, a client is likely to undergo hormone replacement therapy. A transsphenoidal adenohypophysectomy is performed to treat which type of cancer?
You selected:
Pituitary carcinoma
Correct

Explanation:
Pituitary carcinoma most commonly arises in the anterior pituitary (adenohypophysis) and must be removed by way of a transsphenoidal approach, using a bivalve speculum and rongeur.... (more)
Pituitary carcinoma most commonly arises in the anterior pituitary (adenohypophysis) and must be removed by way of a transsphenoidal approach, using a bivalve speculum and rongeur. Surgery to treat esophageal carcinoma usually is palliative and involves esophagogastrectomy with jejunostomy. Laryngeal carcinoma may necessitate a laryngectomy. To treat colorectal cancer, the surgeon removes the tumor and any adjacent tissues and lymph nodes that contain cancer cells. (less)

Question 12:
(see full question)
A nurse is working with a dying client and the client's family. Which communication technique is most important to use?
You selected:
Use active listening and silence when communicating.
Correct

Explanation:
When working with a dying client and the client's family, the nurse should use active listening and silence to assess their feelings, coping skills, and immediate and long-term nee... (more)
When working with a dying client and the client's family, the nurse should use active listening and silence to assess their feelings, coping skills, and immediate and long-term needs. Active listening also helps the nurse select other appropriate strategies, such as reflection and clarification. Interpretation should be used sparingly to avoid making false inferences or putting the client or family on the defensive. Initiate the conversation whenever possible and assess the family and client's coping mechanisms, including what has worked for them in the past. If the nurse is uncertain how to respond, the nurse should ask for more information or clarification from the family not avoid speaking to them. (less)

Question 13:
(see full question)
A right orchiectomy is performed on a client with a testicular malignancy. The client expresses concerns regarding his sexuality. The nurse should base the response on the knowledge that the client:
You selected:
Should retain normal sexual drive and function.
Correct

Explanation:
Unilateral orchiectomy alone does not result in impotence if the other testis is normal. The other testis should produce enough testosterone to maintain normal sexual drive, functi... (more)
Unilateral orchiectomy alone does not result in impotence if the other testis is normal. The other testis should produce enough testosterone to maintain normal sexual drive, functioning, and characteristics. Sperm banking before treatment is commonly recommended because radiation or chemotherapy can affect fertility. (less)

Question 14:
(see full question)
The nurse is encouraging a nursing assistant to interact with a dying client and family. The nurse should help the nursing assistant understand that:
You selected:
When health care personnel do not understand their own feelings about death and dying, they often avoid the client.
Correct

Explanation:
Health care personnel may avoid the terminally ill client because they are uncomfortable about death and do not understand their own feelings about dying. Family members should not... (more)
Health care personnel may avoid the terminally ill client because they are uncomfortable about death and do not understand their own feelings about dying. Family members should not be expected to assume responsibility for the client’s care, but they should be involved in the client’s care to the extent they desire. Skilled and knowledgeable nursing care is required to make a dying person comfortable. Interrupting the client does not necessarily interfere with the right to die with dignity. (less)

Question 15:
(see full question)
A client who has had a total laryngectomy appears withdrawn and depressed. He keeps the curtain drawn, refuses visitors, and indicates a desire to be left alone. Which nursing intervention would most likely be therapeutic for the client?
You selected:
Encouraging him to express his feelings nonverbally and in writing.
Correct

Explanation:
The client has undergone body changes and permanent loss of verbal communication. He may feel isolated and insecure. The nurse can encourage him to express his feelings and use thi... (more)
The client has undergone body changes and permanent loss of verbal communication. He may feel isolated and insecure. The nurse can encourage him to express his feelings and use this information to develop an appropriate plan of care. Discussing the client's behavior with his wife may not reveal his feelings. Exploring future plans is not appropriate at this time because more information about the client's behavior is needed before proceeding to this level. The nurse can respect the client's need for privacy while also encouraging him to express his feelings. (less)

Question 16:
(see full question)
The nurse is reviewing the lab report for a client with breast cancer and brain metastasis. According to the information in the chart, what should the nurse do next?
You selected:
Report the elevated calcium level immediately.
Correct

Explanation:
The normal calcium level is 9.0 to 10.5 mg/dl. Hypercalcemia is commonly seen with malignant disease and metastases. The other laboratory values are normal. Hypercalcemia can be tre... (more)
The normal calcium level is 9.0 to 10.5 mg/dl. Hypercalcemia is commonly seen with malignant disease and metastases. The other laboratory values are normal. Hypercalcemia can be treated with fluids, furosemide (Lasix), or administration of calcitonin. Failure to treat hypercalcemia can cause muscle weakness, changes in level of consciousness, nausea, vomiting, abdominal pain, and dehydration. Although the client is on hospice care, she will still need palliative treatment. Comfort and risk reduction are components of hospice care. (less)

Question 17:
(see full question)
After a lobectomy for lung cancer, the nurse instructs the client to perform deep-breathing exercises to:
You selected:
Expand the alveoli and increase lung surface available for ventilation.
Correct

Explanation:
Deep breathing helps prevent microatelectasis and pneumonitis and also helps force air and fluid out of the pleural space into the chest tubes. It does not decrease blood flow to t... (more)
Deep breathing helps prevent microatelectasis and pneumonitis and also helps force air and fluid out of the pleural space into the chest tubes. It does not decrease blood flow to the lungs or control the rate of air flow. The diaphragm is the major muscle of respiration; deep breathing causes it to descend, thereby increasing the ventilating surface. (less)

Question 18:
(see full question)
A client diagnosed with terminal lung cancer tells the nurse that he would like to seek spiritual advice. Which intervention by the nurse best provides spiritual support for this client?
You selected:
The nurse could ask the client who his spiritual advisor is and make the contact.
Correct

Explanation:
The nurse may contact the client's spiritual advisor if he so desires. The nurse can listen to the client herself, but spiritual support is best from someone proficient in that fie... (more)
The nurse may contact the client's spiritual advisor if he so desires. The nurse can listen to the client herself, but spiritual support is best from someone proficient in that field, such as a spiritual advisor. It would be appropriate for the nurse to contact the clergy of another faith, only if no other resources are available and if the client consents. The nurse should speak with the client and get the information from him firsthand, before researching the admission history. (less)

Question 19:
(see full question)
A nurse is caring for a client who had a prostatectomy for prostate cancer. The nurse is reviewing the client's vital signs and intake and output as documented by a nursing assistant. Which documented finding requires immediate action?
You selected:
Intake and output
Correct

Explanation:
The client has a significantly greater intake than output. This finding may indicate that the catheter is blocked and causing urine retention. The nurse should immediately irrigate... (more)
The client has a significantly greater intake than output. This finding may indicate that the catheter is blocked and causing urine retention. The nurse should immediately irrigate the catheter and try to determine if clots are blocking the catheter. If the nurse is unable to irrigate the catheter, the physician should be notified immediately. The client's heart rate and blood pressure are normal. Although the temperature is slightly elevated, this finding is not a priority at this time. (less)

Question 20:
(see full question)
A client had a total abdominal hysterectomy and bilateral oophorectomy for ovarian carcinoma yesterday. She received 2 mg of morphine sulfate I.V. by patient-controlled analgesia (PCA) 10 minutes ago. The nurse was assisting her from the bed to a chair when the client felt dizzy and fell into the chair. The nurse should:
You selected:
Take the client's blood pressure.
Correct

Explanation:
The nurse should take the client's blood pressure. She is likely experiencing orthostatic hypotension. The PCA pump does not need to be discontinued because, as soon as the blood p... (more)
The nurse should take the client's blood pressure. She is likely experiencing orthostatic hypotension. The PCA pump does not need to be discontinued because, as soon as the blood pressure stabilizes, the pain medication can be resumed. Administering oxygen is not necessary unless the oxygen saturation also drops. The client should sit in the chair until the blood pressure stabilizes. (less)

Question 21:
(see full question)
Hydromorphone 3 mg I.V. has been ordered for a client who is experiencing severe pain. The dosage available is 3 mg/ml. The nurse dilutes the hydromorphone in 5 ml of normal saline solution and prepares to administer the medication via I.V. push over 5 minutes. How many milliliters will the nurse need to administer per minute to administer the drug over 5 minutes?
You selected:
1
Incorrect

Correct response:
1.2
Explanation:
The nurse would need to administer 1.2 ml/min to give the hydromorphone over 5 minutes. When the 3 mg of hydromorphone is diluted with the 5 ml of normal saline solution, a total of... (more)
The nurse would need to administer 1.2 ml/min to give the hydromorphone over 5 minutes. When the 3 mg of hydromorphone is diluted with the 5 ml of normal saline solution, a total of 6 ml of solution results. 6 ml/5 min = 1.2 ml/min. (less)

Question 22:
(see full question)
A 45-year-old single mother of three teenaged boys has metastatic breast cancer. Her parents live 750 miles (1,207 km) away and have only been able to visit twice since her initial diagnosis 14 months ago. The progression of her disease has forced the client to consider high-dose chemotherapy. She is concerned about her children's welfare during the treatment. When assessing the client's present support systems, the nurse will be most concerned about the potential problems with:
You selected:
Support systems and coping strategies.
Correct

Explanation:
The client's resources for coping with the emotional and practical needs of herself and her family need to be assessed because usual coping strategies and support systems are often... (more)
The client's resources for coping with the emotional and practical needs of herself and her family need to be assessed because usual coping strategies and support systems are often inadequate in especially stressful situations. The nurse may be concerned with the client's use of denial, decision-making abilities, and ability to pay for transportation; however, the client's support systems will be of more importance in this situation. (less)

Question 23:
(see full question)
During chemotherapy, an oncology client has a nursing diagnosis of Impaired oral mucous membrane related to decreased nutrition and immunosuppression secondary to the cytotoxic effects of chemotherapy. Which nursing intervention is most likely to decrease the pain of stomatitis?
You selected:
Providing a solution of viscous lidocaine (Xylocaine) for use as a mouth rinse
Correct

Explanation:
To decrease the pain of stomatitis, the nurse should provide a solution of hydrogen viscous lidocaine for the client to use as a mouth rinse. (Commercially prepared mouthwashes con... (more)
To decrease the pain of stomatitis, the nurse should provide a solution of hydrogen viscous lidocaine for the client to use as a mouth rinse. (Commercially prepared mouthwashes contain alcohol and may cause dryness and irritation of the oral mucosa.) The nurse also may administer systemic analgesics as ordered. Stomatitis occurs 7 to 10 days after chemotherapy begins; thus, stopping chemotherapy wouldn't be helpful or practical. Instead, the nurse should stay alert for this potential problem to ensure prompt treatment. Monitoring platelet and leukocyte counts may help prevent bleeding and infection but wouldn't decrease pain in this highly susceptible client. Checking for signs and symptoms of stomatitis also wouldn't decrease the pain. (less)

Question 24:
(see full question)
Which of the following statements is most accurate regarding the long-term toxic effects of cancer treatments on the immune system?
You selected:
The use of radiation and combination chemotherapy can result in more frequent and more severe immune system impairment.
Correct

Explanation:
Studies of long-term immunologic effects in clients treated for leukemia, Hodgkin's disease, and breast cancer reveal that combination treatments of chemotherapy and radiation can ... (more)
Studies of long-term immunologic effects in clients treated for leukemia, Hodgkin's disease, and breast cancer reveal that combination treatments of chemotherapy and radiation can cause overall bone marrow suppression, decreased leukocyte counts, and profound immunosuppression. Persistent and severe immunologic impairment may follow radiation and chemotherapy (especially multiagent therapy). There is no evidence of greater risk of infection in clients with persistent immunologic abnormalities. Suppressor T cells recover more rapidly than the helper T cells. (less)

Question 25:
(see full question)
A client tells the nurse that she has found a painless lump in her right breast during her monthly self-examination. Which assessment finding would strongly suggest that this client's lump is cancerous?
You selected:
Nonmobile mass with irregular edges
Correct

Explanation:
Breast cancer tumors are fixed, hard, and poorly delineated with irregular edges. A mobile mass that is soft and easily delineated is most commonly a fluid-filled benign cyst. Axil... (more)
Breast cancer tumors are fixed, hard, and poorly delineated with irregular edges. A mobile mass that is soft and easily delineated is most commonly a fluid-filled benign cyst. Axillary lymph nodes may or may not be palpable on initial detection of a cancerous mass. Nipple retraction — not eversion — may be a sign of cancer. (less)

Question 26:
(see full question)
An adult is dying from metastatic lung cancer, and all treatments have been discontinued. The client's breathing pattern is labored, with gurgling sounds. The client's spouse asks the nurse, "Can't you do something to help with the breathing?" Which of the following is the nurse's best response in this situation?
You selected:
Reposition the client, elevate the head of the bed, and provide a cool compress.
Correct

Explanation:
Repositioning the client, elevating the head of the bed, and providing a cool compress are comfort interventions consistent with the concept of palliative care of the dying. Direct... (more)
Repositioning the client, elevating the head of the bed, and providing a cool compress are comfort interventions consistent with the concept of palliative care of the dying. Directing the unlicensed personnel to assess vital signs focuses on the dying process, not the client. Suctioning may not benefit the client and is considered invasive and uncomfortable. Telling the spouse an intervention is not needed discounts the spouse's judgment and concerns. (less)

Question 27:
(see full question)
A nurse is caring for a client with a long-term central venous catheter. Which steps should the nurse include in teaching how to care for his catheter at home?
You selected:
Flush each port using a 10ml NSS syringe, giving each port 5ml from the syringe.
Incorrect

Correct response:
Clean the port with an alcohol pad before administering I.V. fluid through the catheter.
Explanation:
Clients should be instructed to clean the port with an alcohol pad before administering I.V. fluid through the catheter to prevent microorganisms from entering the bloodstream. Usi... (more)
Clients should be instructed to clean the port with an alcohol pad before administering I.V. fluid through the catheter to prevent microorganisms from entering the bloodstream. Using clean technique when accessing the port with a needle, cleaning the needle with a povidone-iodine solution, or flushing each port using the same syringe would break sterile technique. (less)

Question 28:
(see full question)
A client with metastatic brain cancer is admitted to the oncology floor. According to the Self-Determination Act of 1991 concerning the execution of an advance directive, the hospital is required to:
You selected: inform the client or legal guardian of their rights to execute an advance directive.
Correct

Explanation:
The client Self-Determination Act of 1991 requires all health care facilities to notify clients upon admission of their right to execute an advance directive. The facility's ethics... (more)
The client Self-Determination Act of 1991 requires all health care facilities to notify clients upon admission of their right to execute an advance directive. The facility's ethics committee can decide on a treatment plan if the client is unable and a health care power of attorney hasn't been appointed. Hospital employees aren't required by law to respect an individual's moral rights; however, the health care professional should respect the client's individual rights as part of his professional responsibility. Health care professionals are sometimes concerned that advance directives prevent treatment that might help the client. However, the hospital isn't required to advise clients not to execute their advance directive. (less)

Question 29:
(see full question)
A nurse is providing breast cancer education at a community facility. The American Cancer Society (Canadian Cancer Society) recommends that women get mammograms:
You selected: yearly starting at age 40.
Correct

Explanation:
The American Cancer Society (Canadian Cancer Society) recommends a mammogram yearly for women age 40 and older. It's recommended that women between ages 20 and 39 have a profession... (more)
The American Cancer Society (Canadian Cancer Society) recommends a mammogram yearly for women age 40 and older. It's recommended that women between ages 20 and 39 have a professional breast examination (not a mammogram) every 3 years. (less)

Question 30:
(see full question)
A nurse is assessing a client with multiple myeloma. The nurse should keep in mind that clients with multiple myeloma are at risk for:
You selected: pathologic bone fractures.
Correct

Explanation:
Clients with multiple myeloma are at risk for pathologic bone fractures secondary to diffuse osteoporosis and osteolytic lesions. Also, clients are at risk for renal failure second... (more)
Clients with multiple myeloma are at risk for pathologic bone fractures secondary to diffuse osteoporosis and osteolytic lesions. Also, clients are at risk for renal failure secondary to myeloma proteins by causing renal tubular obstruction. Liver failure and heart failure aren't usually sequelae of multiple myeloma. Hypoxemia isn't usually related to multiple myeloma. (less)

Question 31:
(see full question)
A nurse is caring for a client who has just had a modified radical mastectomy with immediate reconstruction. She's in her 30s and has two young children. Although she's worried about her future, she seems to be adjusting well to her diagnosis. What should the nurse do to support her coping?
You selected:
Refer the client to a community support program.
Correct

Explanation:
The client isn't withdrawn and doesn't show other signs of anxiety or depression. Therefore, the nurse can probably safely approach her about talking with others who have had simil... (more)
The client isn't withdrawn and doesn't show other signs of anxiety or depression. Therefore, the nurse can probably safely approach her about talking with others who have had similar experiences, a formal community support group. The nurse may educate the client's spouse or partner and listen to his concerns, but the nurse shouldn't tell the client's spouse what to do. The client must consult with her physician and make her own decisions about further treatment. The client needs to express her sadness, frustration, and fear. She can't be expected to be cheerful at all times. (less)

Question 32:
(see full question)
The nurse is conducting an initial nursing history of a client who is experiencing pain related to bone cancer. The most important information to gather in this initial assessment is the:
You selected:
Client’s self-reporting of the pain experience.
Correct

Explanation:
The most important component of pain assessment is the client’s self-report of the pain. The nurse should have the client describe the quality, location, and intensity of the pai... (more)
The most important component of pain assessment is the client’s self-report of the pain. The nurse should have the client describe the quality, location, and intensity of the pain; the client’s response to the pain; and any alleviating or aggravating factors affecting the pain. The physical assessment should follow the pain assessment and should be delayed if the client is uncomfortable. The family’s response to the client’s illness casts light on the amount of support the client has and alerts the nurse to potential problems. With care, however, these concerns are secondary to the issue of pain control. (less)

Question 33:
(see full question)
A nurse is caring for a client receiving chemotherapy. Which assessment finding places the client at the greatest risk for an infection?
You selected:
Stage 3 pressure ulcer on the left heel
Correct

Explanation:
A stage 3 pressure ulcer is a break in the skin's protective barrier, which could lead to infection in a client who is receiving chemotherapy. The WBC count is within normal limits... (more)
A stage 3 pressure ulcer is a break in the skin's protective barrier, which could lead to infection in a client who is receiving chemotherapy. The WBC count is within normal limits. The temperature is slightly elevated, but not as relevant as the pressure ulcer. It is common to report fatigue while receiving chemotherapy. (less)

Question 34:
(see full question)
After receiving chemotherapy for lung cancer, a client's platelet count falls to 98,000/?l. What term should the nurse use to describe this low platelet count?
You selected:
Thrombocytopenia
Correct

Explanation:
A normal platelet count is 140,000 to 400,000/?l in adults. Chemotherapeutic agents produce bone marrow depression, resulting in reduced red blood cell counts (anemia), reduced whit... (more)
A normal platelet count is 140,000 to 400,000/?l in adults. Chemotherapeutic agents produce bone marrow depression, resulting in reduced red blood cell counts (anemia), reduced white blood cell counts (leukopenia), and reduced platelet counts (thrombocytopenia). Neutropenia is the presence of an abnormally reduced number of neutrophils in the blood and is caused by bone marrow depression induced by chemotherapeutic agents. (less)

Question 35:
(see full question)
An adult client with Hodgkin's disease, who weighs 143 lb (65 kg), is to receive vincristine 25 mcg/kg I.V. What is the correct dose in micrograms that the client should receive? Record your answer using a whole number.
You selected:
1625
Correct

Explanation:
Multiply the weight in kilograms by the number of micrograms desired per kilogram:
65 kg × 25 mcg = 1,625 mcg.... (more)
Multiply the weight in kilograms by the number of micrograms desired per kilogram:
65 kg × 25 mcg = 1,625 mcg. (less)

Question 36:
(see full question)
A client receiving 5-fluorouracil is experiencing nausea and vomiting. Which is the nurse’s best course of action?
You selected:
Administer odansetron (Zofran) prior to administering the 5-fluorouracil
Correct

Explanation:
Fluorouracil, an antimetabolite antineoplastic medication, may cause nausea, vomiting, diarrhea, bone marrow suppression, and stomatitis. Premedication with an antiemetic medicatio... (more)
Fluorouracil, an antimetabolite antineoplastic medication, may cause nausea, vomiting, diarrhea, bone marrow suppression, and stomatitis. Premedication with an antiemetic medication such as odansetron will prevent nausea and vomiting during treatment. (less)

Question 37:
(see full question)
The nurse is conducting a community presentation on the early detection of colon cancer. Which of the following should the nurse encourage members of the audience to report to their health care providers? Select all that apply.
You selected:
• Unexplained weight loss with adequate nutritional intake.
• Bowel changes.
• Positive fecal occult blood testing.
Incorrect

Correct response:
• Fatigue.
• Unexplained weight loss with adequate nutritional intake.
• Rectal bleeding.
• Bowel changes.
• Positive fecal occult blood testing.
Explanation:
Colorectal cancer may be asymptomatic, or symptoms vary according to the location of the tumor and the extent of involvement. Fatigue, weight loss, and iron deficiency anemia, even... (more)
Colorectal cancer may be asymptomatic, or symptoms vary according to the location of the tumor and the extent of involvement. Fatigue, weight loss, and iron deficiency anemia, even without rectal bleeding or bowel changes, should prompt investigation for colorectal cancer. Fecal occult blood testing commonly reveals evidence of carcinoma when the client is otherwise asymptomatic. (less)

Question 38:
(see full question)
A 52-year-old male was discharged from the hospital for cancer-related pain. His pain appeared to be well controlled on the I.V. morphine. He was switched to oral morphine when discharged 2 days ago. He now reports his pain as an 8 on a 10-point scale and wants the I.V. morphine. Which of the following represents the most likely explanation for the client's reports of inadequate pain control?
You selected:
He is undermedicated on the oral opioid.
Correct

Explanation:
Most cancer clients with inadequate pain control while taking an oral opioid after being switched from I.V. administration have been undermedicated. Equianalgesic conversions shoul... (more)
Most cancer clients with inadequate pain control while taking an oral opioid after being switched from I.V. administration have been undermedicated. Equianalgesic conversions should be made to provide estimates of the equivalent dose needed for the same level of relief as provided by the I.V. dose. There is research to suggest that cancer clients do not become addicted to opioids when dosed adequately. There is no evidence to suggest that the client is physically addicted or is having withdrawal symptoms. (less)

Question 39:
(see full question)
A client has been receiving chemotherapy for cancer treatment. The client is competent and has been actively involved in decisions regarding care; however, the client has now decided to refuse treatment. What should the nurse do when the client refuses the next dose of chemotherapy?
You selected:
Ensure that the client understands the rationale for taking the medication.
Incorrect

Correct response:
Document the client’s choice and offer to discuss feelings about the chemotherapy.
Explanation:
The client has the right to choose whether to take the medication. The nurse should try to determine the reason for the client not wanting the medication other than choice (e.g., s... (more)
The client has the right to choose whether to take the medication. The nurse should try to determine the reason for the client not wanting the medication other than choice (e.g., side effects, fear of falling asleep and not waking). The other options do not allow for client choice or consent. (less)

Question 40:
(see full question)
The nurse should consider which of the following principles when developing a plan of care to manage a client's pain from cancer?
You selected:
Individualize the pain medication regimen for the client.
Correct

Explanation:
The nurse should work with the client to individualize the plan of care for managing pain. Cancer pain is best managed with a combination of medications, and each client needs to b... (more)
The nurse should work with the client to individualize the plan of care for managing pain. Cancer pain is best managed with a combination of medications, and each client needs to be worked with individually to find the treatment regimen that works best. Cancer pain is commonly undertreated because of fear of addiction. The client who is in pain needs the appropriate level of analgesic and needs to be reassured that he will not become addicted. Cancer pain is best treated with regularly scheduled doses of medication. Administering the medication only when the client asks for it will not lead to adequate pain control. As drug tolerance develops, the dosage of the medication can be increased. (less)

Question 41:
(see full question)
After surgery for gastric cancer, a client is scheduled to undergo radiation therapy. It will be most important for the nurse to include information about which of the following in the client's teaching plan?
You selected:
Nutritional intake.
Correct

Explanation:
Clients who have had gastric surgery are prone to postoperative complications, such as dumping syndrome and postprandial hypoglycemia, that can affect nutritional intake. Vitamin a... (more)
Clients who have had gastric surgery are prone to postoperative complications, such as dumping syndrome and postprandial hypoglycemia, that can affect nutritional intake. Vitamin absorption can also be an issue, depending on the extent of the gastric surgery. Radiation therapy to the upper gastrointestinal area also can affect nutritional intake by causing anorexia, nausea, and esophagitis. The client would not be expected to develop alopecia. Exercise and activity levels as well as access to community resources are important teaching areas, but nutritional intake is a priority need. (less)

Question 42:
(see full question)
Which of the following clinical manifestations does the nurse most likely observe in a client with Hodgkin's disease?
You selected:
Painless, enlarged cervical lymph nodes.
Correct

Explanation:
Painless and enlarged cervical lymph nodes, tachycardia, weight loss, weakness and fatigue, and night sweats are signs of Hodgkin's disease. Difficulty swallowing and breathing may... (more)
Painless and enlarged cervical lymph nodes, tachycardia, weight loss, weakness and fatigue, and night sweats are signs of Hodgkin's disease. Difficulty swallowing and breathing may occur, but only with mediastinal node involvement. Hepatomegaly is a late-stage manifestation. (less)

Question 43:
(see full question)
During a breast examination, which finding most strongly suggests that a client has breast cancer?
You selected:
A fixed nodular mass with dimpling of the overlying skin
Correct

Explanation:
A fixed nodular mass with dimpling of the overlying skin is common during late stages of breast cancer. Many women have slightly asymmetrical breasts. Bloody nipple discharge is a ... (more)
A fixed nodular mass with dimpling of the overlying skin is common during late stages of breast cancer. Many women have slightly asymmetrical breasts. Bloody nipple discharge is a sign of intraductal papilloma, a benign condition. Multiple firm, round, freely movable masses that change with the menstrual cycle indicate fibrocystic breasts, a benign condition. (less)

Question 44:
(see full question)
Which of the following goals is appropriate for a client with multiple myeloma?
You selected:
Achieve effective management of bone pain.
Correct

Explanation:
In multiple myeloma, neoplastic plasma cells invade the bone marrow and begin to destroy the bone. As a result of this skeletal destruction, pain can be significant. There is no cu... (more)
In multiple myeloma, neoplastic plasma cells invade the bone marrow and begin to destroy the bone. As a result of this skeletal destruction, pain can be significant. There is no cure for multiple myeloma. Nausea and vomiting are not characteristics of the disease, although the client may experience anorexia. The client should be monitored for signs of hypercalcemia resulting from bone destruction, not for hyperkalemia. (less)

Question 45:
(see full question)
After lobectomy for lung cancer, a client receives a chest tube connected to a disposable chest drainage system. The nurse observes that the drainage system is functioning correctly when she notes tidal movements or fluctuations in which compartment of the system as the client breathes?
You selected:
Water-seal chamber
Correct

Explanation:
Fluctuations in the water-seal compartment are called tidal movements and indicate normal function of the system as the pressure in the tubing changes with the client's respiration... (more)
Fluctuations in the water-seal compartment are called tidal movements and indicate normal function of the system as the pressure in the tubing changes with the client's respirations. The air-leak meter — not chamber — detects air leaking from the pleural space. The collection chamber connects the chest tube from the client to the system. Drainage from the tube drains into and collects in a series of calibrated columns in this chamber. The suction control chamber provides the suction, which can be controlled to provide negative pressure to the chest. (less)

Question 46:
(see full question)
A client is admitted to the oncology unit with an infection. It is suspected that the infection may be related to their vascular access device (VAD). The nurse should draw the blood cultures from which of the following sites?
You selected:
The proximal lumen of the VAD only.
Incorrect

Correct response:
A peripheral site and all lumens of the VAD.
Explanation:
When an infection is suspected from a VAD, blood cultures should be drawn peripherally and from all lumens of the VAD to determine the source of the infection. If the number of org... (more)
When an infection is suspected from a VAD, blood cultures should be drawn peripherally and from all lumens of the VAD to determine the source of the infection. If the number of organisms is greater from the VAD than in the peripheral culture, the source is determined to be the VAD. (less)

Question 47:
(see full question)
In providing discharge teaching for the client after a modified radical mastectomy, the nurse should instruct the client to avoid which of the following activities?
You selected:
Taking a late-evening swim.
Incorrect

Correct response:
Working in her rose garden.
Explanation:
After a mastectomy, every effort should be made to avoid cuts, bruises, and burns on the affected arm because normal circulation has been impaired. Working in a rose or cactus gard... (more)
After a mastectomy, every effort should be made to avoid cuts, bruises, and burns on the affected arm because normal circulation has been impaired. Working in a rose or cactus garden is a risk because of the danger of skin pricks. The client should be advised to wear protective clothing to prevent cuts, bruises, and burns. As long as it doesn’t increase the risk of injury, caring for the dog is not contraindicated. Maintaining the fish is not contraindicated. Swimming is not contraindicated for the postmastectomy client. (less)

Question 48:
(see full question)
A client is diagnosed with esophageal cancer and presents with difficulty swallowing. Which intervention should receive the highest priority?
You selected:
Ensuring adequate nutrition given the client's recent weight loss of 20 lbs (9.07 kg)
Incorrect

Correct response:
Establishing aspiration precautions
Explanation:
Rapid growth of cancer cells in the esophagus may put pressure on the adjacent trachea, jeopardizing the airway. Therefore, establishing aspiration precautions to protect the airwa... (more)
Rapid growth of cancer cells in the esophagus may put pressure on the adjacent trachea, jeopardizing the airway. Therefore, establishing aspiration precautions to protect the airway is the highest care priority for a client with esophageal cancer. Helping the client cope with body image changes, ensuring adequate nutrition, and providing preoperative teaching for tracheostomy are appropriate for a client with this disease, but are less crucial than airway maintenance. (less)

Question 49:
(see full question)
The nurse is developing a care plan for a client with leukemia. The plan should include which of the following? Select all that apply.
You selected:
• Avoid crowds.
• Monitor temperature and report elevation.
• Recognize signs and symptoms of infection.
• Maintain integrity of skin and mucous membranes.
Correct

Explanation:
Nursing care of a client with leukemia includes managing and preventing infection, maintaining integrity of skin and mucous membranes, instituting measures to prevent bleeding, and... (more)
Nursing care of a client with leukemia includes managing and preventing infection, maintaining integrity of skin and mucous membranes, instituting measures to prevent bleeding, and monitoring for bleeding. Aspirin is an anticoagulant; bleeding tendencies, such as petechiae, ecchymosis, epistaxis, gingival bleeding, and retinal hemorrhages are likely due to thrombocytopenia. (less)

Question 50:
(see full question)
What should a male client older than age 50 do to help ensure early identification of prostate cancer?
You selected:
Perform monthly testicular self-examinations, especially after age 50.
Incorrect

Correct response:
Have a digital rectal examination and prostate-specific antigen (PSA) test done yearly.
Explanation:
The incidence of prostate cancer increases after age 50. The digital rectal examination, which identifies enlargement or irregularity of the prostate, and PSA test, a tumor marker ... (more)
The incidence of prostate cancer increases after age 50. The digital rectal examination, which identifies enlargement or irregularity of the prostate, and PSA test, a tumor marker for prostate cancer, are effective diagnostic measures that should be done yearly. Testicular self-examinations won't identify changes in the prostate gland because of its location in the body. A transrectal ultrasound, CBC, and BUN and creatinine levels are usually done after diagnosis to identify the extent of the disease and potential metastasis. (less)

Hi Christina My Classes Help Center Log Out
1. How Am I Doing?
2. Assignments
3. Practice Quiz
NCLEX-RN
| Med/Surg NCLEX Success Plan
Quiz Results
Quiz Stats
Quiz completed in:
73 min
Total Questions:
50
Questions answered:
50
Number correct:
46
92%

Next

Take another quiz to work towards a higher mastery level.

See your overall performance.

Performance by Chapter
Chapter
Mastery 1
2
3
4
5
6
7
8
Oncologic DisordersOncologic Disorders
3 Quizzes taken
My Mastery Level: 4.00
Class Average: 4.00

Answer Key
Question 1:
(see full question)
A client with cancer is being evaluated for possible metastasis. What is one of the most common metastasis sites for cancer cells?
You selected:
Liver
Correct

Explanation:
The liver is one of the five most common cancer metastasis sites. The others are the lymph nodes, lung, bone, and brain. The colon, reproductive tract, and WBCs are occasional meta... (more)
The liver is one of the five most common cancer metastasis sites. The others are the lymph nodes, lung, bone, and brain. The colon, reproductive tract, and WBCs are occasional metastasis sites. (less)

Question 2:
(see full question)
A client with suspected cervical cancer is undergoing a colposcopy with conization. The nurse gives instructions to the client about her menstrual periods, emphasizing that:
You selected:
Her next two or three periods may be heavier and more prolonged than usual.
Correct

Explanation:
The client should be informed that her next two or three periods could be heavy and prolonged. The client is instructed to report any excessive bleeding. The nurse should reinforce... (more)
The client should be informed that her next two or three periods could be heavy and prolonged. The client is instructed to report any excessive bleeding. The nurse should reinforce the necessity for the follow-up check and the review of the biopsy results with the client. The client's periods will not be normal for 2 to 3 months. (less)

Question 3:
(see full question)
Which of the following measures is most important for pain management for a client after a lobectomy for lung cancer?
You selected:
Reassess the client after administering pain medication.
Correct

Explanation:
It is essential for the nurse to evaluate the effects of pain medication after it has had time to act. Although other interventions may be appropriate, continual reassessment is mo... (more)
It is essential for the nurse to evaluate the effects of pain medication after it has had time to act. Although other interventions may be appropriate, continual reassessment is most important to determine the effectiveness and need for additional intervention, if any. Repositioning could provide some comfort, but assessment of the client's pain level is essential. Reassuring the client is important, but it will be of no value unless the nurse evaluates the client's pain level. To readjust the pain dosage is appropriate only if titration is prescribed by the physician. (less)

Question 4:
(see full question)
Which finding is an early indicator of bladder cancer?
You selected:
Painless hematuria
Correct

Explanation:
Initially, as cancer cells destroy normal bladder tissue, bleeding occurs and causes painless hematuria. (Pain is a late symptom of bladder cancer.) Occasional polyuria may occur w... (more)
Initially, as cancer cells destroy normal bladder tissue, bleeding occurs and causes painless hematuria. (Pain is a late symptom of bladder cancer.) Occasional polyuria may occur with diabetes mellitus or increased alcohol or caffeine intake. Nocturia commonly accompanies benign prostatic hypertrophy. Dysuria may indicate a urinary tract infection. (less)

Question 5:
(see full question)
A client who is in the end-stages of cancer is increasingly upset about receiving chemotherapy. Which of the following approaches by the nurse would likely be most helpful in gaining the client's cooperation?
You selected:
Telling the client how the treatment can be expected to help.
Correct

Explanation:
The best course of action when the client has outbursts concerning treatments is to explain how the treatment is expected to help. Describing the effect if the client misses a trea... (more)
The best course of action when the client has outbursts concerning treatments is to explain how the treatment is expected to help. Describing the effect if the client misses a treatment is a negative approach and may be threatening to the client. The client is likely to feel angry if told to be a "good client" during treatments. Offering to give the client a backrub does not give information to the client and may negatively reinforce the behavior. (less)

Question 6:
(see full question)
Which of the following statements is most accurate regarding the long-term toxic effects of cancer treatments on the immune system?
You selected:
The use of radiation and combination chemotherapy can result in more frequent and more severe immune system impairment.
Correct

Explanation:
Studies of long-term immunologic effects in clients treated for leukemia, Hodgkin's disease, and breast cancer reveal that combination treatments of chemotherapy and radiation can ... (more)
Studies of long-term immunologic effects in clients treated for leukemia, Hodgkin's disease, and breast cancer reveal that combination treatments of chemotherapy and radiation can cause overall bone marrow suppression, decreased leukocyte counts, and profound immunosuppression. Persistent and severe immunologic impairment may follow radiation and chemotherapy (especially multiagent therapy). There is no evidence of greater risk of infection in clients with persistent immunologic abnormalities. Suppressor T cells recover more rapidly than the helper T cells. (less)

Question 7:
(see full question)
A few minutes after beginning a blood transfusion, a nurse notes that a client has chills, dyspnea, and urticaria. The nurse reports this to the physician immediately because the client probably is experiencing which problem?
You selected:
A hemolytic allergic reaction caused by an antigen reaction
Correct

Explanation:
Hemolytic allergic reactions are fairly common and may cause chills, fever, urticaria, tachycardia, dyspnea, chest pain, hypotension, and other signs of anaphylaxis a few minutes a... (more)
Hemolytic allergic reactions are fairly common and may cause chills, fever, urticaria, tachycardia, dyspnea, chest pain, hypotension, and other signs of anaphylaxis a few minutes after blood transfusion begins. Although rare, a hemolytic reaction to mismatched blood can occur, triggering a more severe reaction and, possibly, leading to disseminated intravascular coagulation. A hemolytic reaction to Rh-incompatible blood is less severe and occurs several days to 2 weeks after the transfusion. Bacterial contamination of donor blood causes a high fever, nausea, vomiting, diarrhea, abdominal cramps and, possibly, shock. (less)

Question 8:
(see full question)
A client is diagnosed with esophageal cancer and presents with difficulty swallowing. Which intervention should receive the highest priority?
You selected:
Establishing aspiration precautions
Correct

Explanation:
Rapid growth of cancer cells in the esophagus may put pressure on the adjacent trachea, jeopardizing the airway. Therefore, establishing aspiration precautions to protect the airwa... (more)
Rapid growth of cancer cells in the esophagus may put pressure on the adjacent trachea, jeopardizing the airway. Therefore, establishing aspiration precautions to protect the airway is the highest care priority for a client with esophageal cancer. Helping the client cope with body image changes, ensuring adequate nutrition, and providing preoperative teaching for tracheostomy are appropriate for a client with this disease, but are less crucial than airway maintenance. (less)

Question 9:
(see full question)
A client has undergone surgical resection for lung cancer. Which of the following nursing interventions will promote adaptation and rehabilitation?
You selected:
Planning a progressive activity regimen with the client.
Correct

Explanation:
A progressive activity regimen may be prescribed to increase pulmonary function after surgical lung resection. Rehabilitation should include walking and some stair climbing as tole... (more)
A progressive activity regimen may be prescribed to increase pulmonary function after surgical lung resection. Rehabilitation should include walking and some stair climbing as tolerated. Vigorous exercise is usually not recommended initially. Joining the Lost Chord Club (Canadian Lung Cancer Society) and learning tracheostomy care are appropriate for the client who has undergone a laryngectomy. (less)

Question 10:
(see full question)
A client diagnosed with terminal lung cancer tells the nurse that he would like to seek spiritual advice. Which intervention by the nurse best provides spiritual support for this client?
You selected:
The nurse could ask the client who his spiritual advisor is and make the contact.
Correct

Explanation:
The nurse may contact the client's spiritual advisor if he so desires. The nurse can listen to the client herself, but spiritual support is best from someone proficient in that fie... (more)
The nurse may contact the client's spiritual advisor if he so desires. The nurse can listen to the client herself, but spiritual support is best from someone proficient in that field, such as a spiritual advisor. It would be appropriate for the nurse to contact the clergy of another faith, only if no other resources are available and if the client consents. The nurse should speak with the client and get the information from him firsthand, before researching the admission history. (less)

Question 11:
(see full question)
A nurse is checking the laboratory results of an adult client with colon cancer admitted for further chemotherapy. The client has lost 30 lb (13.6 kg) since initiation of the treatment. Which laboratory result should be reported to the health care provider?
You selected:
Albumin level of 2.8 g/dl (28 g/L).
Correct

Explanation:
The nurse must recognize that an albumin level of 2.8 g/dl (28 g/L) indicates catabolism and potential for malnutrition. Normal albumin is 3.5 to 5.0 g/dl (30 to 50 g/L); less than 3.5... (more)
The nurse must recognize that an albumin level of 2.8 g/dl (28 g/L) indicates catabolism and potential for malnutrition. Normal albumin is 3.5 to 5.0 g/dl (30 to 50 g/L); less than 3.5 g/dl (35 g/L) indicates malnutrition. The other laboratory results are normal. (less)

Question 12:
(see full question)
The nurse explains to the client with Hodgkin's disease that a bone marrow biopsy will be taken after the aspiration. What should the nurse explain about the biopsy?
You selected:
"You may hear a crunch as the needle passes through the bone, but when the biopsy is taken, you will feel a suction-type pain that will last for just a moment."
Incorrect

Correct response:
"You will feel a pressure sensation when the biopsy is taken but should not feel actual pain; if you do, tell the doctor so that you can be given extra numbing medicine."
Explanation:
A biopsy needle is inserted through a separate incision in the anesthetized area. The client will feel a pressure sensation when the biopsy is taken but should not feel actual pain... (more)
A biopsy needle is inserted through a separate incision in the anesthetized area. The client will feel a pressure sensation when the biopsy is taken but should not feel actual pain. The client should be instructed to inform the physician if pain is felt so that more anesthetic agent can be administered to keep the client comfortable. The biopsy is performed after the aspiration and from a slightly different site so that the tissue is not disturbed by either test. The client will feel a suction-type pain for a moment when the aspiration is being performed, not the biopsy. A small incision is made for the biopsy to accommodate the larger-bore needle. This may require a stitch. (less)

Question 13:
(see full question)
Which of the following characteristics displayed by the wife of a 36-year-old man with pancreatic cancer suggests that she may be at risk for negative bereavement outcomes?
You selected:
She blames herself for her husband's cancer.
Correct

Explanation:
Variables that are most predictive of negative bereavement outcomes include anger and self-reproach, low socioeconomic status, lack of preparation for death, and lack of family sup... (more)
Variables that are most predictive of negative bereavement outcomes include anger and self-reproach, low socioeconomic status, lack of preparation for death, and lack of family support. Making preparations suggests that she is coping with her husband's approaching death. (less)

Question 14:
(see full question)
During a breast examination, which finding most strongly suggests that a client has breast cancer?
You selected:
A fixed nodular mass with dimpling of the overlying skin
Correct

Explanation:
A fixed nodular mass with dimpling of the overlying skin is common during late stages of breast cancer. Many women have slightly asymmetrical breasts. Bloody nipple discharge is a ... (more)
A fixed nodular mass with dimpling of the overlying skin is common during late stages of breast cancer. Many women have slightly asymmetrical breasts. Bloody nipple discharge is a sign of intraductal papilloma, a benign condition. Multiple firm, round, freely movable masses that change with the menstrual cycle indicate fibrocystic breasts, a benign condition. (less)

Question 15:
(see full question)
What should a male client older than age 50 do to help ensure early identification of prostate cancer?
You selected:
Have a digital rectal examination and prostate-specific antigen (PSA) test done yearly.
Correct

Explanation:
The incidence of prostate cancer increases after age 50. The digital rectal examination, which identifies enlargement or irregularity of the prostate, and PSA test, a tumor marker ... (more)
The incidence of prostate cancer increases after age 50. The digital rectal examination, which identifies enlargement or irregularity of the prostate, and PSA test, a tumor marker for prostate cancer, are effective diagnostic measures that should be done yearly. Testicular self-examinations won't identify changes in the prostate gland because of its location in the body. A transrectal ultrasound, CBC, and BUN and creatinine levels are usually done after diagnosis to identify the extent of the disease and potential metastasis. (less)

Question 16:
(see full question)
When explaining hospice care to a client, which of the following statements is most appropriate?
You selected:
"Clients and their families are the focus of care."
Correct

Explanation:
The most important central component of hospice care is focus of care on the client as well as the family or significant other. The team approach and the physician's coordination o... (more)
The most important central component of hospice care is focus of care on the client as well as the family or significant other. The team approach and the physician's coordination of the hospice team are important, but they are not the focus. Not all hospice clients want to die at home. (less)

Question 17:
(see full question)
A nurse is performing a home visit for a client who received chemotherapy within the past 24 hours. The nurse observes a small child playing in the bathroom, where the toilet lid has been left up. Based on these observations, the nurse modifies the client's teaching plan to include:
You selected: chemotherapy exposure and risk factors.
Correct

Explanation:
The raised toilet lid exposes the child playing in the bathroom to the risk of inhaling or ingesting chemotherapy agents. The nurse should modify her teaching plan to include conte... (more)
The raised toilet lid exposes the child playing in the bathroom to the risk of inhaling or ingesting chemotherapy agents. The nurse should modify her teaching plan to include content related to chemotherapy exposure and its associated risk factors. Because the client has received chemotherapy, the plan should already include information about expected adverse effects, signs and symptoms of infection, and reinforcement of the medication regimen. (less)

Question 18:
(see full question)
A client with a diagnosis of cancer is frequently disruptive and challenges the nurse. This behavior may be caused by:
You selected:
Uncertainty and an underlying fear of recurrence.
Correct

Explanation:
Cancer clients report that the lifelong fear of recurrence is one of the most disruptive aspects of the disease. The trajectory of the disease is unpredictable and can be intertwin... (more)
Cancer clients report that the lifelong fear of recurrence is one of the most disruptive aspects of the disease. The trajectory of the disease is unpredictable and can be intertwined with many short- and long-term illnesses related to cancer and the treatment modalities. A diagnosis of cancer challenges the individual and the family with a series of crises rather than a time-limited episode. There are no data to indicate that the client has an underlying behavioral disorder. (less)

Question 19:
(see full question)
After surgery for gastric cancer, a client is scheduled to undergo radiation therapy. It will be most important for the nurse to include information about which of the following in the client's teaching plan?
You selected:
Nutritional intake.
Correct

Explanation:
Clients who have had gastric surgery are prone to postoperative complications, such as dumping syndrome and postprandial hypoglycemia, that can affect nutritional intake. Vitamin a... (more)
Clients who have had gastric surgery are prone to postoperative complications, such as dumping syndrome and postprandial hypoglycemia, that can affect nutritional intake. Vitamin absorption can also be an issue, depending on the extent of the gastric surgery. Radiation therapy to the upper gastrointestinal area also can affect nutritional intake by causing anorexia, nausea, and esophagitis. The client would not be expected to develop alopecia. Exercise and activity levels as well as access to community resources are important teaching areas, but nutritional intake is a priority need. (less)

Question 20:
(see full question)
Prior to surgery for a modified radical mastectomy, the client is extremely anxious and asks many questions. Which of the following approaches would offer the best guide for the nurse to answer questions raised by this apprehensive preoperative client?
You selected:
Tell the client as much as she wants to know and is able to understand.
Correct

Explanation:
An important nursing responsibility is preoperative teaching, and the most frequently recommended guide for teaching is to tell the client as much as she wants to know and is able ... (more)
An important nursing responsibility is preoperative teaching, and the most frequently recommended guide for teaching is to tell the client as much as she wants to know and is able to understand. Delaying discussion of issues about which the client has concerns is likely to aggravate the situation and cause the client to feel distrust. As a general guide, the client would not ask the question if she were not ready to discuss her situation. The nurse is available to answer the client's questions and concerns and should not delay discussing these with the client. (less)

Question 21:
(see full question)
Which of the following clients is at highest risk for colorectal cancer?
You selected:
The client who smokes.
Incorrect

Correct response:
The client who has been treated for Crohn's disease for 20 years.
Explanation:
Clients over age 50 who have a history of inflammatory bowel disease are at risk for colon cancer. The client who smokes is at high risk for lung cancer. While the exact cause is n... (more)
Clients over age 50 who have a history of inflammatory bowel disease are at risk for colon cancer. The client who smokes is at high risk for lung cancer. While the exact cause is not always known, other risk factors for colon cancer are a diet high in animal fats, including a large amount of red meat and fatty foods with low fiber, and the presence of colon cancer in a first-generation relative. (less)

Question 22:
(see full question)
A client undergoes a biopsy of a suspicious lesion. The biopsy report classifies the lesion according to the TNM staging system as follows: TIS, N0, M0. What does this classification mean?
You selected:
Carcinoma in situ, no abnormal regional lymph nodes, and no evidence of distant metastasis
Correct

Explanation:
TIS, N0, M0 denotes carcinoma in situ, no abnormal regional lymph nodes, and no evidence of distant metastasis. No evidence of primary tumor, no abnormal regional lymph nodes, and ... (more)
TIS, N0, M0 denotes carcinoma in situ, no abnormal regional lymph nodes, and no evidence of distant metastasis. No evidence of primary tumor, no abnormal regional lymph nodes, and no evidence of distant metastasis is classified as T0, N0, M0. If the tumor and regional lymph nodes can't be assessed and no evidence of metastasis exists, the lesion is classified as TX, NX, M0. A progressive increase in tumor size, no demonstrable metastasis of the regional lymph nodes, and ascending degrees of distant metastasis is classified as T1, T2, T3, or T4; N0; and M1, M2, or M3. (less)

Question 23:
(see full question)
During chemotherapy, an oncology client has a nursing diagnosis of Impaired oral mucous membrane related to decreased nutrition and immunosuppression secondary to the cytotoxic effects of chemotherapy. Which nursing intervention is most likely to decrease the pain of stomatitis?
You selected:
Providing a solution of viscous lidocaine (Xylocaine) for use as a mouth rinse
Correct

Explanation:
To decrease the pain of stomatitis, the nurse should provide a solution of hydrogen viscous lidocaine for the client to use as a mouth rinse. (Commercially prepared mouthwashes con... (more)
To decrease the pain of stomatitis, the nurse should provide a solution of hydrogen viscous lidocaine for the client to use as a mouth rinse. (Commercially prepared mouthwashes contain alcohol and may cause dryness and irritation of the oral mucosa.) The nurse also may administer systemic analgesics as ordered. Stomatitis occurs 7 to 10 days after chemotherapy begins; thus, stopping chemotherapy wouldn't be helpful or practical. Instead, the nurse should stay alert for this potential problem to ensure prompt treatment. Monitoring platelet and leukocyte counts may help prevent bleeding and infection but wouldn't decrease pain in this highly susceptible client. Checking for signs and symptoms of stomatitis also wouldn't decrease the pain. (less)

Question 24:
(see full question)
The nurse is reviewing the lab report shown for a client with breast cancer and brain metastasis. According to the information in the chart, what should the nurse do next?
You selected:
Report the elevated calcium level immediately.
Correct

Explanation:
The normal calcium level is 9.0 to 10.5 mg/dL (2.3 to 2.6 mmol/L). Hypercalcemia is commonly seen with malignant disease and metastases. The other laboratory values are normal. Hyper... (more)
The normal calcium level is 9.0 to 10.5 mg/dL (2.3 to 2.6 mmol/L). Hypercalcemia is commonly seen with malignant disease and metastases. The other laboratory values are normal. Hypercalcemia can be treated with fluids, furosemide, or administration of calcitonin. Failure to treat hypercalcemia can cause muscle weakness, changes in consciousness, nausea, vomiting, abdominal pain, and dehydration. Although the client is on hospice care, she will still need palliative treatment. Comfort and risk reduction are components of hospice care. (less)

Question 25:
(see full question)
The nurse is assessing a 42-year-old client with cancer who has lost 1 lb (0.45 kg) in 4 weeks. The client is taking ondansetron for nausea and now has a temperature of 101 degrees F (38.3 degrees C). The fever is indicative of:
You selected:
Infection.
Correct

Explanation:
Fever is most commonly related to infection. In a neutropenic client, fever frequently occurs in the absence of the usual clinical signs and symptoms of infection. Inadequate nutri... (more)
Fever is most commonly related to infection. In a neutropenic client, fever frequently occurs in the absence of the usual clinical signs and symptoms of infection. Inadequate nutrition or antiemetic therapy resistance would not result in fever. Fever is not usually expected with most chemotherapy drugs. (less)

Question 26:
(see full question)
The client asks the nurse to explain what it means that his Hodgkin's disease is diagnosed at stage 1A. Which of the following describes the involvement of the disease?
You selected:
Involvement of a single lymph node.
Correct

Explanation:
In the staging process, the designations A and B signify, respectively, that symptoms were or were not present when Hodgkin's disease was found. The Roman numerals I through IV ind... (more)
In the staging process, the designations A and B signify, respectively, that symptoms were or were not present when Hodgkin's disease was found. The Roman numerals I through IV indicate the extent and location of involvement of the disease. Stage I indicates involvement of a single lymph node; stage II, two or more lymph nodes on the same side of the diaphragm; stage III, lymph node regions on both sides of the diaphragm; and stage IV, diffuse disease of one or more extralymphatic organs. (less)

Question 27:
(see full question)
A client has been receiving chemotherapy to treat cancer. Which assessment finding suggests that the client has developed stomatitis?
You selected:
Red, open sores on the oral mucosa
Correct

Explanation:
The tissue-destructive effects of cancer chemotherapy typically cause stomatitis, resulting in ulcers on the oral mucosa that appear as red, open sores. White, cottage cheese–lik... (more)
The tissue-destructive effects of cancer chemotherapy typically cause stomatitis, resulting in ulcers on the oral mucosa that appear as red, open sores. White, cottage cheese–like patches on the tongue suggest a candidal infection, another common adverse effect of chemotherapy. Yellow tooth discoloration may result from antibiotic therapy, not cancer chemotherapy. Rust-colored sputum suggests a respiratory disorder, such as pneumonia. (less)

Question 28:
(see full question)
A 76-year-old client tells the nurse that she has lived long and does not need mammograms. Which is the nurse's best response?
You selected:
"The incidence of breast cancer increases with age."
Correct

Explanation:
Advancing age in postmenopausal women has been identified as a risk factor for breast cancer. A 76-year-old client needs monthly breast self-examination and a yearly clinical breas... (more)
Advancing age in postmenopausal women has been identified as a risk factor for breast cancer. A 76-year-old client needs monthly breast self-examination and a yearly clinical breast examination and mammogram to comply with the screening schedule. While mammograms are less painful as breast tissue becomes softer, the nurse should advise the woman to have the mammogram. Family history is important, but only about 5% of breast cancers are genetic. (less)

Question 29:
(see full question)
The nurse at the gynecologic clinic is teaching the client about the results of her Papanicolaou test, which demonstrated dysplasia. Which represents the nurse’s best intervention?
You selected:
Explain the results show alteration in the size and shape of cells, which requires follow-up.
Correct

Explanation:
Dysplasia, a precancerous condition, refers to an alteration in the size, shape, and organization of differentiated cells. The client will need further diagnostic evaluation to det... (more)
Dysplasia, a precancerous condition, refers to an alteration in the size, shape, and organization of differentiated cells. The client will need further diagnostic evaluation to determine scope and treatment of the problem. The other options are not the best interventions. (less)

Question 30:
(see full question)
A client, age 42, visits the gynecologist. After examining the client, the physician suspects cervical cancer. The nurse reviews the client's history for risk factors for this disease. Which history finding is a risk factor for cervical cancer?
You selected:
Human papillomavirus infection at age 32
Correct

Explanation:
Like other viral and bacterial venereal infections, human papillomavirus is a risk factor for cervical cancer. Other risk factors for this disease include frequent sexual intercour... (more)
Like other viral and bacterial venereal infections, human papillomavirus is a risk factor for cervical cancer. Other risk factors for this disease include frequent sexual intercourse before age 20, multiple sex partners, and multiple pregnancies. A spontaneous abortion and pregnancy complicated by eclampsia aren't risk factors for cervical cancer. (less)

Question 31:
(see full question)
After a lobectomy for lung cancer, the nurse instructs the client to perform deep-breathing exercises to:
You selected:
Expand the alveoli and increase lung surface available for ventilation.
Correct

Explanation:
Deep breathing helps prevent microatelectasis and pneumonitis and also helps force air and fluid out of the pleural space into the chest tubes. It does not decrease blood flow to t... (more)
Deep breathing helps prevent microatelectasis and pneumonitis and also helps force air and fluid out of the pleural space into the chest tubes. It does not decrease blood flow to the lungs or control the rate of air flow. The diaphragm is the major muscle of respiration; deep breathing causes it to descend, thereby increasing the ventilating surface. (less)

Question 32:
(see full question)
Which of the following positions is appropriate for palpating tissues during breast self-examination?
You selected:
Standing facing a mirror.
Incorrect

Correct response:
Flat on the back with a pillow under the shoulder of the side being examined.
Explanation:
For a breast self-examination, placing a pillow or towel under the shoulder of the side being examined elevates the chest wall while the woman lies flat on her back. This positioni... (more)
For a breast self-examination, placing a pillow or towel under the shoulder of the side being examined elevates the chest wall while the woman lies flat on her back. This positioning allows for better breast tissue distribution over the chest wall and the most thorough examination of tissues by palpation. Sitting is not the desired position for palpating the breasts. A standing position, facing a mirror, is used to inspect the breasts for changes in size and shape, skin dimpling, and nipple changes, but not for palpation. Flat on the back with a pillow under the head and arms raised over the head is not a recommended position as it cannot expose the tissue better than raising only one arm with a pillow under the shoulder. (less)

Question 33:
(see full question)
The nurse is conducting an initial nursing history of a client who is experiencing pain related to bone cancer. The most important information to gather in this initial assessment is the:
You selected:
Client’s self-reporting of the pain experience.
Correct

Explanation:
The most important component of pain assessment is the client’s self-report of the pain. The nurse should have the client describe the quality, location, and intensity of the pai... (more)
The most important component of pain assessment is the client’s self-report of the pain. The nurse should have the client describe the quality, location, and intensity of the pain; the client’s response to the pain; and any alleviating or aggravating factors affecting the pain. The physical assessment should follow the pain assessment and should be delayed if the client is uncomfortable. The family’s response to the client’s illness casts light on the amount of support the client has and alerts the nurse to potential problems. With care, however, these concerns are secondary to the issue of pain control. (less)

Question 34:
(see full question)
A client's bone marrow report reveals normal stem cells and precursors of platelets (megakaryocytes) in the presence of decreased circulating platelets. The nurse recognizes a knowledge deficit when the client makes which of the following statements?
You selected:
"I am glad that my report turned out normal."
Correct

Explanation:
The client who states that the test results are normal has only heard that the bone marrow is functioning. The etiology is in the destruction of circulating platelets. Further test... (more)
The client who states that the test results are normal has only heard that the bone marrow is functioning. The etiology is in the destruction of circulating platelets. Further tests must be completed to determine the cause (e.g., a coating of the platelets with antibodies that are seen as foreign bodies). The bone marrow result does rule out other potential diagnoses such as anemia, leukemia, or myeloproliferative disorders that involve bone marrow depression. The client needs to stop flossing and throw away his hard toothbrush, which can lead to bleeding of the gums. The destruction of the circulating platelets accounts for the easy bruising and the need to protect oneself from further bruising. The client should not jump or increase exertion of joints, which may lead to bleeding in the joints and joint pain. (less)

Question 35:
(see full question)
A 30-year-old client whose mother died of breast cancer at age 44 and whose sister has ovarian cancer, is concerned about developing cancer. As a member of the oncology multidisciplinary team, the nurse should suggest that the client ask the physician about which topic?
You selected:
Papanicolaou (Pap) testing every 6 months
Incorrect

Correct response:
Genetic counseling
Explanation:
The nurse should suggest that the client ask the physician about genetic counseling. Genetic counseling is indicated for those at high risk because of family or personal cancer his... (more)
The nurse should suggest that the client ask the physician about genetic counseling. Genetic counseling is indicated for those at high risk because of family or personal cancer history. Genetic counseling involves obtaining a detailed medical and three-generational family history; calculating a personalized risk assessment; providing options for prevention, surveillance, and genetic testing; coordinating and interpreting genetic testing; and developing a management plan based on the test results. Mammography will assist with early detection of most breast cancers, but it won't establish a risk assessment and provide options for prevention, surveillance, and genetic testing. Pap testing every 6 months assists in early detection of most cervical cancers, but it won't establish a risk assessment. Contacting the American Cancer Society (Canadian Cancer Society) won't help assess the client's risk for developing cancer. (less)

Question 36:
(see full question)
A client has a routine Papanicolaou (Pap) test during a yearly gynecologic examination. The result reveals a class V finding. What should the nurse tell the client about this finding?
You selected:
It calls for a biopsy as soon as possible.
Correct

Explanation:
A class V finding in a Pap test suggests probable cervical cancer; the client should have a biopsy as soon as possible. Only a class I finding, which is normal, requires no action.... (more)
A class V finding in a Pap test suggests probable cervical cancer; the client should have a biopsy as soon as possible. Only a class I finding, which is normal, requires no action. A class II finding, which indicates inflammation, calls for a repeat Pap test in 3 months. A class III finding, which indicates mild to moderate dysplasia, calls for a repeat Pap test in 6 weeks to 3 months. A class IV finding indicates possible cervical cancer; like a class V finding, it warrants a biopsy as soon as possible. (less)

Question 37:
(see full question)
A client receiving external radiation to the left thorax to treat lung cancer has a nursing diagnosis of Risk for impaired skin integrity. Which intervention should be part of this client's care plan?
You selected:
Avoiding using deodorant soap on the irradiated areas
Correct

Explanation:
Because external radiation commonly causes skin irritation, the nurse should wash the irradiated area with water and a mild soap only and leave the area open to air. No deodorants ... (more)
Because external radiation commonly causes skin irritation, the nurse should wash the irradiated area with water and a mild soap only and leave the area open to air. No deodorants or powders should be applied. A lead apron is unnecessary because no radiation source is present in the client's body or room. Skin in the area to be irradiated is marked to position the radiation beam as precisely as possible; skin markings must not be removed. (less)

Question 38:
(see full question)
After a transsphenoidal adenohypophysectomy, a client is likely to undergo hormone replacement therapy. A transsphenoidal adenohypophysectomy is performed to treat which type of cancer?
You selected:
Pituitary carcinoma
Correct

Explanation:
Pituitary carcinoma most commonly arises in the anterior pituitary (adenohypophysis) and must be removed by way of a transsphenoidal approach, using a bivalve speculum and rongeur.... (more)
Pituitary carcinoma most commonly arises in the anterior pituitary (adenohypophysis) and must be removed by way of a transsphenoidal approach, using a bivalve speculum and rongeur. Surgery to treat esophageal carcinoma usually is palliative and involves esophagogastrectomy with jejunostomy. Laryngeal carcinoma may necessitate a laryngectomy. To treat colorectal cancer, the surgeon removes the tumor and any adjacent tissues and lymph nodes that contain cancer cells. (less)

Question 39:
(see full question)
A client seeks care for hoarseness that has lasted for 1 month. To elicit the most appropriate information about this problem, the nurse should ask which question?
You selected:
"Do you smoke cigarettes, cigars, or a pipe?"
Correct

Explanation:
Persistent hoarseness may signal throat cancer, which commonly is associated with tobacco use. To assess the client's risk for throat cancer, the nurse should ask about smoking hab... (more)
Persistent hoarseness may signal throat cancer, which commonly is associated with tobacco use. To assess the client's risk for throat cancer, the nurse should ask about smoking habits. Although straining the voice may cause hoarseness, it wouldn't cause hoarseness lasting for 1 month. Consuming red meat or spicy foods isn't associated with persistent hoarseness. (less)

Question 40:
(see full question)
Which of the following statements indicates that the client needs further teaching about taking medication to control cancer pain?
You selected:
”I should skip doses periodically so I don’t get hooked on my drugs.”
Correct

Explanation:
The client should not skip his dosages of pain medication to prevent addiction. Clients with cancer pain do not become psychologically dependent on the medication and should not fe... (more)
The client should not skip his dosages of pain medication to prevent addiction. Clients with cancer pain do not become psychologically dependent on the medication and should not fear becoming addicted. The nurse should allow the client and family members to verbalize their concerns about drug addiction. (less)

Question 41:
(see full question)
A client tells the nurse that she has found a painless lump in her right breast during her monthly self-examination. Which assessment finding would strongly suggest that this client's lump is cancerous?
You selected:
Nonmobile mass with irregular edges
Correct

Explanation:
Breast cancer tumors are fixed, hard, and poorly delineated with irregular edges. A mobile mass that is soft and easily delineated is most commonly a fluid-filled benign cyst. Axil... (more)
Breast cancer tumors are fixed, hard, and poorly delineated with irregular edges. A mobile mass that is soft and easily delineated is most commonly a fluid-filled benign cyst. Axillary lymph nodes may or may not be palpable on initial detection of a cancerous mass. Nipple retraction — not eversion — may be a sign of cancer. (less)

Question 42:
(see full question)
For a cancerous tumor, a client must undergo modified radical mastectomy, which includes axillary node removal and immediate reconstruction. The nurse explains to the client that the axillary nodes will be removed to:
You selected: provide prognostic information.
Correct

Explanation:
Lymph node dissection provides prognostic information by helping to determine if chemotherapy is indicated. Although removal of lymph nodes may assist in prevention of metastasis, ... (more)
Lymph node dissection provides prognostic information by helping to determine if chemotherapy is indicated. Although removal of lymph nodes may assist in prevention of metastasis, lymph node dissection isn't a guarantee that metastasis won't occur. This procedure doesn't affect breast reconstruction and may actually make postoperative recovery more difficult. (less)

Question 43:
(see full question)
After lobectomy for lung cancer, a client receives a chest tube connected to a disposable chest drainage system. The nurse observes that the drainage system is functioning correctly when she notes tidal movements or fluctuations in which compartment of the system as the client breathes?
You selected:
Water-seal chamber
Correct

Explanation:
Fluctuations in the water-seal compartment are called tidal movements and indicate normal function of the system as the pressure in the tubing changes with the client's respiration... (more)
Fluctuations in the water-seal compartment are called tidal movements and indicate normal function of the system as the pressure in the tubing changes with the client's respirations. The air-leak meter — not chamber — detects air leaking from the pleural space. The collection chamber connects the chest tube from the client to the system. Drainage from the tube drains into and collects in a series of calibrated columns in this chamber. The suction control chamber provides the suction, which can be controlled to provide negative pressure to the chest. (less)

Question 44:
(see full question)
Experimental and epidemiologic evidence suggests that a high-fat diet increases the risk of several cancers. Which of the following cancers is linked to a high-fat diet?
You selected:
Colon.
Correct

Explanation:
Evidence suggests that a high-fat diet increases the risk of several cancers, including breast, colon, and prostate cancers. Ovarian, lung, and liver cancers have not been linked t... (more)
Evidence suggests that a high-fat diet increases the risk of several cancers, including breast, colon, and prostate cancers. Ovarian, lung, and liver cancers have not been linked to a high-fat diet. (less)

Question 45:
(see full question)
After being seen in the oncology clinic, a client with severe bone marrow suppression is admitted to the hospital. The client's cancer therapy consisted of radiation and chemotherapy. When developing the care plan for this client, the nurse prioritizes which nursing intervention?
You selected:
Monitoring temperature and blood cell count
Correct

Explanation:
Risk for infection takes highest priority in clients with severe bone marrow depression. This is because they have a decrease in the number of white blood cells, which are the cell... (more)
Risk for infection takes highest priority in clients with severe bone marrow depression. This is because they have a decrease in the number of white blood cells, which are the cells that fight infection. Therefore, the nurse should monitor temperature and blood cell count. While the other interventions are helpful in the care of this client, the risk for infection takes precedence. (less)

Question 46:
(see full question)
After a mastectomy for breast cancer, the nurse teaches the client how to avoid the development of lymphedema. Which of the following instructions should be included?
You selected:
Elevating the affected arm on a pillow.
Correct

Explanation:
The client should be taught to elevate the affected arm on a pillow to promote venous return and lymphatic drainage of the area. Applying an elastic bandage is inappropriate becaus... (more)
The client should be taught to elevate the affected arm on a pillow to promote venous return and lymphatic drainage of the area. Applying an elastic bandage is inappropriate because constriction of the extremity should be avoided. Range-of-motion exercising is not limited. Rather, it is encouraged. Diuretics are not used to control lymphedema. (less)

Question 47:
(see full question)
A client with metastatic brain cancer is admitted to the oncology floor. According to the Self-Determination Act of 1991 concerning the execution of an advance directive, the hospital is required to:
You selected: inform the client or legal guardian of their rights to execute an advance directive.
Correct

Explanation:
The client Self-Determination Act of 1991 requires all health care facilities to notify clients upon admission of their right to execute an advance directive. The facility's ethics... (more)
The client Self-Determination Act of 1991 requires all health care facilities to notify clients upon admission of their right to execute an advance directive. The facility's ethics committee can decide on a treatment plan if the client is unable and a health care power of attorney hasn't been appointed. Hospital employees aren't required by law to respect an individual's moral rights; however, the health care professional should respect the client's individual rights as part of his professional responsibility. Health care professionals are sometimes concerned that advance directives prevent treatment that might help the client. However, the hospital isn't required to advise clients not to execute their advance directive. (less)

Question 48:
(see full question)
The client with Hodgkin's disease undergoes an excisional cervical lymph node biopsy under local anesthesia. After the procedure, which does the nurse assess first?
You selected:
The airway.
Correct

Explanation:
Assessing for an open airway is always first. The procedure involves the neck; the anesthesia may have affected the swallowing reflex, or the inflammation may have closed in on the... (more)
Assessing for an open airway is always first. The procedure involves the neck; the anesthesia may have affected the swallowing reflex, or the inflammation may have closed in on the airway, leading to ineffective air exchange. Once a patent airway is confirmed and an effective breathing pattern established, the circulation is checked. Vital signs and the incision are assessed as soon as possible, but only after it is established that the airway is patent and the client is breathing normally. A neurologic assessment is completed as soon as possible after other important assessments. (less)

Question 49:
(see full question)
A nurse is assessing a female who is receiving her second administration of chemotherapy for breast cancer. When obtaining this client's health history, what is the most important information the nurse should obtain?
You selected:
"Have you had nausea or vomiting?"
Correct

Explanation:
Chemotherapy agents typically cause nausea and vomiting when not controlled by antiemetic drugs. Antineoplastic drugs attack rapidly growing normal cells, such as in the gastrointe... (more)
Chemotherapy agents typically cause nausea and vomiting when not controlled by antiemetic drugs. Antineoplastic drugs attack rapidly growing normal cells, such as in the gastrointestinal tract. These drugs also stimulate the vomiting center in the brain. Hair loss, loss of energy, and sleep are important aspects of the health history, but not as critical as the potential for dehydration and electrolyte imbalance caused by nausea and vomiting. (less)

Question 50:
(see full question)
A client is to have radiation therapy after a modified radical mastectomy. The nurse should teach the client to care for the skin at the site of therapy by:
You selected:
Washing the area with water.
Correct

Explanation:
A client receiving radiation therapy should avoid lotions, ointments, and anything that may cause irritation to the skin, such as exposure to sunlight, heat, or talcum powder. The ... (more)
A client receiving radiation therapy should avoid lotions, ointments, and anything that may cause irritation to the skin, such as exposure to sunlight, heat, or talcum powder. The area may safely be washed with water if it is done gently and if care is taken not to injure the skin. (less)

Online Product Support: 1-800-468-1128 | techsupp@lww.com
© 2009-14 PrepU, A division of Macmillan

You May Also Find These Documents Helpful

  • Good Essays

    Ati Questions

    • 527 Words
    • 3 Pages

    2. Which of the following nursing interventions should be implemented to maintain a patent airway in a client on bed rest?…

    • 527 Words
    • 3 Pages
    Good Essays
  • Good Essays

    Med Surg Notes

    • 1606 Words
    • 7 Pages

    1. The nurse checks for placement of a nasogastric tube before beginning tube feeding for a client. Which of the following results would indicate to the RN that the tube feeding can begin?…

    • 1606 Words
    • 7 Pages
    Good Essays
  • Better Essays

    The first priority is to perform a focused assessment to include the patient’s respiratory function, pain, mental status, and any medication the patient has taken. The patient’s airway and ability to breathe and maintain a patent airway becomes the first priority. By asking the patient the four questions of orientation the nurse can assess the patient’s mental status. The patient’s pain can also be assessed quickly by using a numerical value or the Wong-Baker Scale prior to the patient becoming unresponsive, as well as asking the patient for a brief history of her medical condition and any co-morbidities. For the patient’s airway and breathing, the patient should be placed on 15 liters of oxygen with a non-rebreather mask to allow for increased oxygenation and a pulse…

    • 1422 Words
    • 6 Pages
    Better Essays
  • Better Essays

    References: Lewis, S. L., Dirksen, S. R., Heitkemper, M. M., Bucher, L., & Camera, I. M. (2011). Medical-Surgical Nursing: Assessment and Management of Clinical Problems (Eighth Edition). St. Louis, MO: Elsevier Mosby.…

    • 1608 Words
    • 5 Pages
    Better Essays
  • Powerful Essays

    Acute care essay

    • 2487 Words
    • 7 Pages

    On admission to the hospital it is important that Mr Taylor is given an immediate and systematic assessment which will alert the healthcare professionals to any deterioration in his condition. The assessment method used in this case is the Airway, Breathing, Circulation, Disability, Exposure (ABCDE) approach. Thim et al (2012) suggests that the aims of this approach are to provide the patient with lifesaving treatment, the ability to break down into more manageable chunks the complex clinical situation. It also provides all healthcare providers with a common awareness and serves as an algorithm for treatment and assessing which in turn can buy time to enable a final diagnosis and treatment plan to be made. According to the National Institute for Health and Clinical Excellence (NICE) (2007), patients may receive suboptimal care if staff do no act on clinical deterioration in a timely manner.…

    • 2487 Words
    • 7 Pages
    Powerful Essays
  • Good Essays

    5 parts of the run

    • 607 Words
    • 3 Pages

    a. Airway-determine if the airway is open if closed open with head tilt chin lift or in trauma jaw trust maneuver. If the patient is alert and responsive the airway is patent. Assess, intervene, reassess…

    • 607 Words
    • 3 Pages
    Good Essays
  • Good Essays

    Nurse logic

    • 1352 Words
    • 7 Pages

    4. In assessing Josh's breathing sounds, the nurse should ask him to perform which action?…

    • 1352 Words
    • 7 Pages
    Good Essays
  • Good Essays

    Task EBT1

    • 1869 Words
    • 7 Pages

    Standard practice is for the attending nurse of the non-critical hospital patient is to perform a…

    • 1869 Words
    • 7 Pages
    Good Essays
  • Powerful Essays

    Study Guide

    • 2488 Words
    • 10 Pages

    It is important for the nurse to assess respirations of the client when administering opioids because of the possibility of respiratory depression. The other interventions may or may not be necessary in the care of the client and do not focus on safety.…

    • 2488 Words
    • 10 Pages
    Powerful Essays
  • Satisfactory Essays

    Medical Surgical Nursing

    • 695 Words
    • 3 Pages

    1. What first actions should the nurse take after the patient has arrived in the emergency department?…

    • 695 Words
    • 3 Pages
    Satisfactory Essays
  • Good Essays

    Flawless Smile

    • 640 Words
    • 3 Pages

    Chronic tobacco use results in bad breath, gum disease and teeth stains. This is because nicotine turns yellow when mixed with oxygen. Therefore, as the tobacco makes contact with the teeth, it leeches into tiny openings in the enamel. Smokeless tobacco products, such as chewing tobacco, is much worse for the teeth. This is because there are higher levels of nicotine. Chronic use of chewing tobacco will cause the gums to receded and increase the rate of tooth…

    • 640 Words
    • 3 Pages
    Good Essays
  • Satisfactory Essays

    Journal Article Review

    • 636 Words
    • 3 Pages

    All nurses are taught to do respiratory observations on every patient as initial and ongoing routine assessments. Nevertheless, what the author observed in her clinical placement was…

    • 636 Words
    • 3 Pages
    Satisfactory Essays
  • Good Essays

    As of 1996, there were 30,000 new cases of oral cancer diagnosed in the United States, one-third of which were fatal. Some studies estimate that using smokeless tobacco raises the risk of oral cancer, which includes cancers of the throat, by as much as 50 percent. Smokeless tobacco use has also been strongly linked to cancers of the pancreas, nasal cavities, esophagus, pharynx (passageway for food and air), intestines, stomach, larynx ("voice box"), and urinary tract. Other no cancer health problems associated with smokeless tobacco include peptic ulcers, danger to fetuses, and damage of the salivary…

    • 2244 Words
    • 9 Pages
    Good Essays
  • Good Essays

    Today’s clinical experience allowed for new education and skill practice. I was able to precept in post-op, which was a great change. Being in pre-op, we are responsible for receiving clients from the OR. We then monitor them, reeducate, prepare for and perform discharge, and cleaning the area that was used by disinfecting, tossing and replacing linens, and moving the bed to an empty room in pre-op. We receive report from the circulating nurse and the nurse anesthetist when they first bring the patient to recovery. Together we hook the patient up to the monitors and record the first set of vitals together. These include: blood pressure, heart rate, respirations, temperature, pulse ox, pain (if patient is conscious), and an ECG reading if they were general.…

    • 1454 Words
    • 6 Pages
    Good Essays
  • Good Essays

    Essay On First Aid

    • 706 Words
    • 3 Pages

    The first part of the primary survey is always assessing the airway. This includes checking for potential injury to the cervical spine. Until cervical spine injury has been ruled out, open the patient's airway using a jaw-thrust maneuver with manual, in-line stabilization of the neck. If you find food, blood, vomitus, or other debris, suction the airway quickly to prevent aspiration. To better remove secretions, you may need to carefully logroll the patient to his side. Manually stabilize his neck and spine as you do so.…

    • 706 Words
    • 3 Pages
    Good Essays